You are on page 1of 60

SELECCIONE LA MEJOR RESPUESTA CORRECTA PARA CADA ENUNCIADO O

CASO CLNICO PRESENTADO.


RECUERDE RELLENAR CADA CRCULO DE FORMA COMPLETA. SOLO SE PERMITE
EL USO DE LAPIZ GRAFITO SUAVE N 2.
CONTESTE TODO EL EXAMEN. CONTAR CON 4 HORAS RELOJ PARA REALIZAR
LA PRUEBA. NO SE DAR TIEMPO ADICIONAL LUEGO DE COMPLETADO ESTE
TIEMPO. NO PIERDE PUNTOS POR ADIVINAR. NO HABR TIEMPO ADICIONAL
PARA TRASPASAR LAS PREGUNTAS DEL FOLLETO A LA HOJA DE RESPUESTAS.
NO ARRANQUE O DESTRUYA PARTE DEL MANUAL O LA HOJA DE RESPUESTAS.
PUEDE CONSULTAR LA GUA DE VALORES DE LABORATORIOS QUE SE
ENCUENTRA AL FINAL DE LA PRUEBA TODAS LAS VECES QUE SEA NECESARIO
PERO NO PUEDE DESPRENDERLAS DEL FOLLETO
NO PUEDE REALIZAR CONSULTAS SOBRE PREGUNTAS O RESPUESTAS A
COMPAEROS O PROFESORES DURANTE LA PRUEBA
USTED PUEDE DEJAR EL SALN SI HA COMPLETADO LA PRUEBA ANTES DE LOS
LTIMOS 10 MINUTOS DEL TIEMPO ESTIPULADO. SI POR MOTIVOS RELIGIOSOS,
CULTURALES, NECESIDADES FISIOLGICAS REQUIERE SALIR DEL SALN DEL
EXAMEN DEBER LEVANTAR LA MANO PARA SER ACOMPAADO DURANTE ESE
TIEMPO, NO SE LE OTORGAR TIEMPO ADICIONAL PARA COMPENSAR.
NO SE PERMITEN EL USO DE TELFONOS INTELIGENTES, CELULARES, RELOJES
CON CAPACIDAD DE MEMORIA U OTRO TIPO DE AYUDA MANUAL O
ELECTRNICA. SOLO PUEDE TENER SU LAPIZ, SACAPUNTA Y BORRADOR
SOBRE EL ESCRITORIO
LOS ABRIGOS DEBERN ESTAR COLOCADOS Y CERRADOS O DOBLADOS Y
GUARDADOS DEBAJO DE LAS BANCAS. LAS BOLSAS ESTARN CERRADAS Y
UBICADAS EN EL FRENTE DEL SALN.
LOS TELFONOS CELULARES
SILENCIADOS.
UNA VEZ DADO INICIO A LA PRUEBA NADIE PODR INGRESAR AL RECINTO SIN
EVALUACIN DEL JEFE DE REA DE EVALUACIN.
Esta prueba tiene derecho de autor de GrupoMdico Panam,SA la duplicacin,
replicacin se considera desagravio serio y puede ser penalizado.
Firma del estudiante ______________________________________________

1) Femenina de 68 aos de edad que viene a la sala de emergencias con una hernia
umbilical encarcelada evidente. Tiene distensin abdominal grave, clnicamente
deshidratada y reporta vmitos fecaloides persistentes durante los ltimos 3 das. Aunque
cansada, dbil y sedienta, ella esta despierta y alerta y su sensorio no ha sido afectado.
Los laboratorios revelan concentracin srica de sodio de 118 mEq/L. ella no tiene
condiciones mdicas crnicas ni toma medicamentos. Cul de los siguientes es la
explicacin fisiolgica ms probable para el nivel de sodio srico?
A) Ella tiene intoxicacin aguda de agua
B) Ella ha estado vomitando y atrapando fluidos hipertnicos en el lumen intestinal
C) Ella ha vomitado y secuestrado fluidos que contiene sodio, y ha conservado agua
endgena e ingesta
D) Debe de ser un error de laboratorio, tal nivel de sodio srico producira coma
E) Depravacin de volumen conduce a perdida renal de sodio
2) Femenina de 82 aos que se presenta a sala de emergencias con dolor precordial. Ella
se despert temprano esta maana con un dolor precordial centralizado, sordo e
inmediatamente marco el 911, ya que pens que estaba teniendo un ataque cardiaco.
Afirma que ha tenido episodios similares antes, pero que ningn haba sido tan severo.
Comenz con dolores precordiales hace dos semanas, especficamente cuando camina
alrededor del centro comercial. Fue una fumadora de 45 paquetes por ao, pero dejo de
fumar hace 10 aos despus de la muerte de su esposo debido a infarto agudo de
miocardio. Signos vitales, con temperatura de 37,2C (98,9F), pulso de 115/min, Presin
arterial de 150/90 mmHg. Al examen fsico, paciente aparenta adolorida y est sudando
profusamente. Actualmente est respirando 100% de oxigeno por la mscara facial. No
tiene distensin yugular. Sonidos cardiacos distantes, pero regulares en ritmo y
frecuencia. Un EKG no revela elevacin del segmento ST. Ella comenz con aspirina,
enoxaparina, atenolol, nitratos y morfina. Cul es el siguiente paso en el manejo?
A) Adicionar atorvastatina
B) Adicionar clopidogrel
C) Adicionar enalapril
D) Intervencin coronaria percutnea
E) Medicin de troponina y CK-MB
3) Femenina caucsica de 19 aos que acude porque ha dejado de tomar en los ltimos
dos meses sus pastillas anticonceptivas y una prueba de embarazo de orina resulto
positiva. No recuerda cuando fue su ltimo periodo. Se siente bien y no aqueja nada. Su
historia mdica pasada sin significancia. Vive con su madre y dos hermanos. No toma, no
bebe o usa drogas ilcitas. Su temperatura es de 37C (98,6F), Presin arterial de 104/78
mmHg, pulso de 72/min, y Fr de 16/min. Examen fsico revela sensibilidad mamaria, tero
grvido, crvix teido de azul, y es de otra manera sin complicaciones. USG muestra un
embarazo intrauterino de 12 semanas. Laboratorios prenatales son analizados, y el frotis
de PAP y orina y cultivo de crvix son obtenidos. Un urinlisis muestra bacteriuria
moderada con estereasa leucocitaria y nitritos positiva. Se le administro folato y vitaminas
prenatales. Cul es el siguiente mejor paso en el manejo?
A) No es necesario un manejo adicional como pacente asintomtico
B) TMP/SMX
C) Nitrofurantoina

D) Levonogestrel
E) Miconazole
4) Femenina de 45 aos en sala de emergencias por aturdimiento despus de un largo
paseo en carro. Su esposo reporta que no ha tomado fluidos, usualmente toma de 20-24
bebidas sin alcohol diariamente, y orina frecuentemente. Presin arterial de 110/75 mmHg
cuando esta supina y 88/60 al sentarse, pulso de 80/min supina y de 102/min al sentarse.
Laboratorios muestran: Glucosa en orina negativo
Osmolaridad urinaria 110 mosmol/Kg
Osmolaridad urinaria es de 176 despus de administrar vasopresina.
Cul de los siguientes es el diagnstico ms probable?
A) Diabetes inspida central
B) Toma compulsiva de agua
C) Diabetes mellitus 1
D) Diabetes mellitus 2
E) Diabetes inspida Neurognica
5) Nio de 4 aos es tradO a la clnica peditrica debido a debilidad y malestar. l ha
reportado a su madre que estos sntomas se presentaron desde el ltimo mes. Ella ha
notado tambin que cuando lo baa, lo siente inusualmente rgido. l se niega a comer
tanto como lo sola hacer y no ha estado ganando peso. El tambin aqueja sensacin de
mareo y dolor en el vientre. Sin antecedentes mdicos relevantes. Tiene todas sus
inmunizaciones al da. Al examen fsico, esta afebril con Presin arterial de 80/48 mmHg.
Es encontrado con hipotensin ortosttica. El resto del examen sin alteraciones.
Laboratorios:
Glbulos blancos: 6,8/mm3
Plaquetas: 252,000/mm3
Sodio: 130 mEq/L
Cloro: 97 mEq/L
BUN: 14 mg/dL

Hematocrito: 46%
Glucosa: 70 mg/dL
Potasio: 5,6 mEq/L
Bicarbonato: 24 mEq/L
Creatinina: 0,6 mg/dL

Una prueba de estimulacin demuestra respuesta subptima de ACTH. Un ensayo de


cidos grasos de cadena larga demuestra altos niveles en la sangre del nio. Cul de los
siguientes es caracterstico de este diagnstico?
A) Trierucato de glicerol (aceite de Lorenzo) se ha demostrado que retrasa la progresin
de los sntomas una vez ellos han comenzado
B) Es un desorden ligada a X
C) Imagen de RM del cerebro es usualmente normal
D) Hallazgos neurolgicos usualmente precede al diagnstico de insuficiencia adrenal
E) El gen responsable es una protena de la membrana mitocondrial
6) Chico de 17 aos vienen por historia de fiebre de 10 das, malestar, dolor de garganta.
El tomo medicamentos de resfriado sin mejora de los sntomas. Tomo alguna ampicilina
sobrante que encontr en la cabina de medicamentos hace 7 das. Ha tenido sarpullido
rojo en los ltimos 4 das. Es un chico muy activo en el deporte y ha tenido que perderse 2
juegos de basketball en la pasada semana. Sin relevancia en historia mdica. Es
sexualmente activo y usa condn consistentemente. Su temperatura es de 38,7C
3

(101,6F). Al examinarlo presenta amgdala eritematosa y edematosa con exudado claro.


Presenta ganglios linfticos submandibulares agrandados y sensibles. Examen cutneo
muestra una erupcin maculopapular y eritematoso en el tronco y extremidades. El borde
esplnico se palpa 2 cm por debajo del margen costal. Una prueba rpida de estreptococo
es negativa. Cul de los siguientes es la causa ms probable de la condicin del
paciente?
A) Estreptococo beta hemoltico de grupo A
B) Virus del Epstein Barr
C) Virus herpes simplex
D) VIH
E) Rubivirus
7) Nio de 5 aos es visto en visita rutinaria de salud, y se encuentra en crecimiento
adecuado. Est cumpliendo con el desarrollo de todos los hitos y generalmente esta
saludable. Al examen fsico, tiene apariencia masculina saludable, con el borde heptico
3cm debajo del margen costal. Laboratorios realizados con resultados:
Glbulos blancos
3,670/mm3 Hb
10,3g/dL
Hematocrito 36%
Plaquetas
130,000/mm3. Una radiografa abdominal confirma aumento de tamao de
hgado y bazo, y muestra osteopenia de cuerpos vertebrales. Una inspeccin de
seguimiento seo muestra resplandecimiento de la porcin tubular inferior de los fmures
distales. Cul sera la causa ms probable de estos hallazgos?
A) Alfa-mannosidosis
B) Enfermedad de Gaucher (GD)
C) Sndrome de Hunter
D) Enfermedad de Niemann Pick tipo A
E) Enfermedad de Sanfillipo tipo B
8) Masculino de 24 aos que viene para seguimiento despus de completar curso de 6
meses de tratamiento para tuberculosis actica que adquiri durante un viaje. Actualmente
se siente bien, y no especifica queja. Entre sus antecedentes mdicos incluyen un nuevo
caso de tratamiento difcil de dermatitis seborreica y un episodio reciente de herpes
zoster. Adems de esas condiciones, el paciente no presenta enfermedades crnicas.
Signos vitales y examen fsico sin complicaciones, con la excepcin de hallazgos
cutneos consistente con dermatitis seborreica. El paciente est preocupado, sin
embargo, acerca de sus posibilidades de ser un caso recurrente de TBC. Cul de los
siguientes es la prueba ms apropiada para ofrecer al paciente?
A) Prueba cutneas de alergia
B) Radiografa de trax
C) Prueba de HIV
D) Colocacin de PPD
E) Cultivo y frotis de esputo
9) masculino de 29 aos de edad que se presenta al doctor debido a que ha notado que
su ojo se ha tornado amarillo. Esto se ha presentado intermitentemente desde hace 2
aos. No presenta ningn otro sntoma y se siente bien. Niega sntomas gripales
recientes, malestar, mialgias o dolor en cuadrante superior derecho. No ha tenido prdida
de peso, distensin abdominal, hematemesis o hemorroides. Es heterosexual
exclusivamente y ha tenido mltiples parejas sexuales. Algunos de los encuentros
sexuales ha sido sin proteccin. Est actualmente soltero. Toma alcohol y fuma
4

socialmente. Su historia familiar es notable solamente por hipertensin y enfermedad


arterial coronaria. Niega uso de drogas intravenosas y nunca ha recibido transfusin. Al
examen fsico, est bien desarrollado y nutrido. Esta afebril y normotenso. Examen de
cabeza y cuello revelan ictericia escleral sin linfadenopata. Examen cutneo no revela
angioma en araa o eritema palmar. No tiene ginecomastia. Examen cardiovascular y
pulmonar normal. Examen abdominal no revela distensin, cabeza de medusa o
sensibilidad en cuadrante superior derecho. Laboratorios:
Creatinina: 0,8 mg/dL
Glucosa: 104 mg/dL
Albumina: 4,1 g/dL
AST: 33 U/L
ALT: 25 U/L
Alcalina fosfatasa: 65 U/L
LDH: 110 U/L
Haptoglobina: 102 mg/dL
Bilirrubina total: 3,2 mg/dL
Bilirrubina conjugada 0,3 mg/dL
Anticuerpos de superficie de hepatitis B: Positivo
IgM anti hepatitis A: Negativo
Anticuerpos anti hepatitis C: Negativo
Frotis de sangre perifrica: Sin clulas anormales
Cul de los siguientes es la explicacin ms probable para la ictericia del paciente?
A) Infeccin aguda de hepatitis B
B) Sndrome de Gilbert
C) Hemlisis
D) Masa en la cabeza del pncreas
D) Cirrosis biliar primaria

10) Femenina de 45 aos es trada a sala de emergencias por su hermana 20 minutos


despus de una convulsin generalizada. La hermana afirma que la paciente ha tenido
episodios intermitentes de sacudidas, palpitacin, sudoracin y ansiedad por 6 meses. No
toma medicamentos. Trabaja como farmacutica. Esta diafortica y alerta. Su temperatura
es de 36,7C (98F), Presin arterial de 130/85 mmHg, pulso de 110/min, anticuerpos
contra insulina negativa. Laboratorios que muestran:
Glucosa sangunea: 45 mg/dL
Insulina: 7 U/mL (VN <3 U/mL)
Pptido C: 1,5 nmol/L (VN <0,2 nmol/L)
Proinsulina: 9 pmol/L (VN <5 pmol/L)
Cul de los siguientes es el diagnstico ms probable?
A) Sndrome de Zollinger Ellison
B) Administracin facticia de insulina
C) Glucagonoma
D) Insulinoma
E) Feocromocitoma
11) Una mujer de 24 aos de edad llega al departamento de emergencia debido a dolor y
visin borrosa en el ojo izquierdo desde hace 2 das. Ha tenido dificultad para respirar
desde hace 2 meses. Su temperatura es de 37.9C, presin arterial de 116/78, y pulso de
90 cpm. El examen fsico muestra ndulos eritematosos y sensibilidad en la cara anterior
de ambas piernas. Los pulmones se encuentran claros a la auscultacin. El examen con
lmpara evidencia fotofobia en ojo izquierdo, eritema alrededor de la crnea
(circumcorneal) y precipitados blanquecinos en la crnea. La presin intraocular es
5

normal. Una radiografa de trax muestra ensanchamiento hiliar bilateral. Cul de las
siguientes caractersticas es ms probable en esta condicin?
A) Diarrea sanguinolenta y tenesmo
B) Cronicidad, curso no-remitente
C) Enfermedad de glndulas partidas y lacrimales
D) Infecciones recurrentes con microorganismos encapsulados
E) Poliartritis simtrica
12) Una mujer de 27 aos de edad es admitida al hospital debido a dolor articular de 4
das de evolucin. Inicialmente tuvo un dolor articular difuso que migr desde sus
muecas hacia sus codos y hacia sus rodillas. Durante las ltimas 24 horas, sin embargo,
ha tenido un intenso dolor e hinchazn localizados en la rodilla derecha. Niega traumas en
el rea. Niega fiebre y escalofros, pero refiere leve disuria que atribuye a una infeccin de
vas urinarias. Su temperatura es de 38.1C, presin arterial 120/70, pulso de 98 cpm,
frecuencia respiratoria de 18 cpm. Al examen fsico su rodilla est hinchada y sensible a la
palpacin. Tiene pstulas necrticas y diseminadas en sus palmas y plantas. Se realiza
una artrocentesis y el frotis Gram muestra diplococos gram negativos.
Cul de los siguientes es el siguiente paso ms apropiado para el manejo?
A) Ceftriaxona IV
B) Nafcilina IV
C) Penicilina IV
D) Vancomicina IV
E) Aguante los antibiticos y espere por el cultivo y resultados de sensibilidad
13) Un hombre de 38 aos con historia de herpes genital, acude al mdico por dolor e
hinchazn de su rodilla derecha. Reporta que sus sntomas inician temprano en el da y le
resulta extremadamente doloroso el movimiento o tocarse la rodilla. Niega trauma en el
rea afectada. Comenta haber tenido relaciones sexuales recientemente sin proteccin
con una prostituta. Su temperatura es de 39C, presin arterial 125/90, pulso regular de
108 cpm y frecuencia respiratoria de 15 cpm. El examen fsico revela una rodilla derecha
sensible a la palpacin e hinchada, con efusin palpable. El paciente se encuentra
taquicrdico, pero por lo dems al examen fsico est normal. Se realiza una artrocentesis
de la rodilla derecha y se enva el fluido para su anlisis. Cul de los siguientes
hallazgos es ms probable?
A) Conteo celular con 75 000 WBC
B) Cultivo positivo para cocos Gram positivos en racimos
C) Frotis Gram que revela diplococos Gram positivos
D) Cristales con birrefigencia negativa
E-) Cristales en forma de rombo
14) Femenina de 34 aos acude al mdico por dolores mensuales por 12 meses. Tambin
ha tenido movimientos intestinales dolorosos, sin sangre en heces. Ella ha sido incapaz
de concebir a pesar de tener relaciones sexuales regulares. Ha estado bajo uso de
medroxiprogesterona por 8 aos pero hace 15 meses que ha dejado de inyectarse, desde
ese tiempo ha tenido periodos regulares. Examen plvico muestra una apariencia normal
de vulva, vagina y crvix. Sin masas anexas. Examen rectovaginal muestra nodulaciones
en ligamento uterosacro con dolor a la palpacin. Estudios de laboratorios muestran:

Hb: 10.3g/dL
Hct: 30.5%
MCV: 74 microm3
Glbulos blancos: 9,300 mm3
Neutrfilos: 56%
Bandas: 3%
Plaquetas: 205,000 mm3
VES (ESR): 68 mm/h
Cul de los siguientes es el sitio anatmico ms comn afectado por esta condicin?
A) Vejiga
B) Ovarios
C) Peritoneo parietal
D) rganos peritoneales
E) Recto
15) masculino de 27 aos que acude a examen fsico anual. Esta asintomtico y al
examinarlo se escucha un murmullo de eyeccin sistlica en el borde izquierdo inferior del
esternn. El murmullo se vuelve ms fuerte cuando se pone de pie y con Valsalva, pero
se reduce su intensidad cuando se agacha. En el EKG muestra una marcada hipertrofia
ventricular izquierda. Cul de los siguientes es el prximo paso ms apropiado en el
manejo?
A) Drogas anti arrtmicas
B) Evitar ejercicio competitivo
C) Referir para un desfibrilador cardiaco automtico implantable
D) Referir para miomectoma quirrgica
E) Comenzar betabloqueadores
16) Femenina de 83 aos de edad que acude porque ella se siente muy cansada todo el
tiempo. Hematocrito en la oficina es 35%. Frotis de sangre perifrica muestra algunos
glbulos rojos macrocticos. En la entrevista, la mujer cuyas finanzas estn limitadas,
admite que ha estado bajo dieta estricta principalmente de pan. Ella ha estado tomando
sustituto de jugo de naranja en polvo. Ella no ha estado tomando vitaminas porque siente
que no puede pagarlas. Una deficiencia nutricional de cul de los siguientes es la causa
ms probable de anemia en este paciente?
A) Folato
B) Hierro
C) Vitamina B12
D) Vitamina C
E) Vitamina K
17) Una mujer de 21 aos, primigesta, con 39 semanas de gestacin, viene a la sala de
labor y parto con contracciones dolorosas cada 3 minutos. Su evolucin prenatal tiene
datos irrelevantes. Al examen muestra un cuello con 3 cm de dilatacin y borramiento de
90%. La FCF en un trazo es de 150 LPM y est reactivo. 5 horas despus, al examinar el
cuello, se evidencia que tiene 9 cm y se encuentra en plano -1. Los trazos de la FCF
muestran desaceleraciones variables moderadas con cada contraccin y variabilidad
disminuida. Un muestreo de cuero cabelludo del feto es realizado y arroja los siguientes
resultados, pH: 7.04, 7.05 y 7.06. Cul de los siguientes es el siguiente paso ms
apropiado en el manejo?
A) Cesrea
B) Episiotoma
C) Manejo conservador
7

D) Parto asistido con frceps


E) Parto asistido con ventosas
18) Un hombre de 61 aos, es trado al cuarto de urgencias por dolor en el pecho. l tiene
una historia de enfermedad coronaria de larga data e hipertensin y se le realiz una
derivacin coronaria hace 6 aos. El paciente tiene una angina estable crnica que es
precipitada usualmente por actividad y que se alivia con el reposo. Su medicacin es
aspirina, captopril y metoprolol. Hace 3 semanas, su mdico le prescribi sildenafil y ha
usado el medicamento con xito. Su ltimo encuentro sexual fue hace 4 das. Esta
maana, ha desarrollado un malestar en el pecho. Sus signos vitales en el cuarto de
urgencias son: Temperatura 36.1C, BP 220/120 mmHg, pulso en 100 por minuto y
frecuencia respiratoria en 22 por minuto. Al examen ocular muestra papiledema, el ECG
revela cambios no especficos. Cul de las siguientes opciones, es el mejor tratamiento
indicado para este momento?
A) Labetalol intravenoso
B) Morfina intravenosa
C) Clonidina oral
D) Metoprolol oral
E) Nitroglicerina oral
19) Una mujer de 29 aos, saludable, estaba subiendo las escaleras en su nueva casa
hace un mes, cuando la casa, colaps. Ella fue trada al hospital con una fractura de
fmur izquierdo. Se le hizo una interconsulta al servicio de psiquiatra porque la paciente
se quejaba de pesadillas y recuerdos y tena miedo de ir a dormir por esto. Durante la
entrevista, ella tena llanto fcil y estaba asustada por su miedo a caer y esto le impide
participar plenamente en su rehabilitacin, por lo cual, ella teme que los mdicos le den
salida. Cul de los siguientes es el tratamiento ms apropiado?
A) Psicoterapia orientada introspectivamente
B) No dar terapias porque la paciente necesita tomar responsabilidad para su tratamiento
C) Colocar a un cuidador que est con la paciente en la sala, las 24 horas para calmar su
ansiedad
D) Iniciar antidepresivos
E) Iniciar benzodiacepinas
20) Un hombre de 43 aos acude al mdico por historia de 4 aos con dolor articular.
Tiene dolor en las pequeas articulaciones proximales y distales de su mano derecha, en
la rodilla izquierda, el tobillo derecho y el codo izquierdo. Ha tenido rigidez matinal que
dura hasta 2 horas. Su historial mdico es significativo solo por unos parches escamosos
en mis codos que nunca han sido atendidos por un mdico. Su temperatura es de 37C,
presin arterial de 120/70, pulso 76 cpm y frecuencia respiratoria de 13 cpm. El examen
fsico muestra deformidades en las articulaciones interfalngicas proximales y distales
(subluxaciones) del dedo ndice, medio y anular de la mano derecha; deformidad
interfalngica proximal del dedo anular de la mano izquierda; todos los dedos de las
manos presentan punteado ungueal (signo de la ua en dedal) y decoloracin de las
uas. Laboratorios: Hb-14.0g/dL; Hematocrito: 42%; glbulos blancos en 5 000/mm3;
Plaquetas: 200 000/mm3; Factor Reumatoideo Negativo y VES: 46 mm/h Cul es el
diagnstico ms probable?
A) Onicomicosis
B) Osteroartritis
8

C) Pseudogota
D) Artriris psorisica
E) Artriris Reactiva
21) Una mujer de 65 aos con cncer metastsico es admitida al hospital por un severo
dolor en su pierna izquierda desde hace 2 semanas. El est en su muslo izquierdo cerca
de su cadera. Ha tenido problemas para caminar debido al dolor. El dolor empeora en la
noche. No tiene fiebre, rigidez matinal, o trauma. La fosfatasa alcalina srica de la
paciente se encuentra en 400 U/L. Una radiografa del muslo muestra una lesin ltica en
su fmur proximal izquierdo. Cul de las siguientes es ms probable que sea la
malignidad de la paciente?
A) Mama
B) Colon
C) Melanoctico
D) Pncreas
E) Tiroides
22) Un hombre de 42 aos acude al mdico por epistaxis, tos productiva de esputo teido
con sangre. Su historial mdico es irrelevante y no toma medicamentos. Su temperatura
es de 37.7C, pulso de 80 cpm. El examen fsico muestra una mucosa nasal edematosa y
una lcera localizada en el septum nasal. Se escuchan crepitantes bilateralmente en los
campos pulmonares. Glbulos blancos en 12 500/mm3. Laboratorio positivo para
Anticuerpos Citiplasmticos Antineutrfilos. El urinlisis muestra sangre microscpica,
cilindros hemticos y 2+ de protenas. Cul es la farmacoterapia ms apropiada?
A) Azatioprina
B) Ciclofosfamida+prednisona
C) Metotrexate
D) Plasmaferesis
E) Prednisona
23) Una mujer de 49 aos acude a su mdico por dificultad para comer desde hace 6
meses. Ella refiere tener sequedad en la boca desde hace 1 ao. Ha dejado de utilizar sus
lentes de contactos por sequedad e irritacin en los ojos. Se le ha diagnosticado 3 caries
en el ao pasado. El examen fsico revela agrandamiento bilateral de las mejillas. La
mucosa oral luce seca y su lengua est atrfica. Tiene la piel seca de manera difusa. Una
biopsia de glndula partida muestra un infiltrado inflamatorio linfoctico. Cul de los
siguientes autoanticuerpos es ms especfico para la condicin del paciente?
A) Anticuerpos anti-pptido citrulinado (Anti-CCP) y anticuerpos anti-IgG
B) Anticuerpos anti-DNA de doble cadena y anticuerpos anti-Ro
C) Anticuerpos anti-IgG y anticuerpos anti-U1-RNP
D) Anticuerpos anti-La y anti-Ro
E) Anticuerpos anti-Smith y anti-SSA
24) Un hombre de 50 aos acude al mdico por dolor articular de 3 meses de duracin. El
dolor va y viene y tpicamente ocurre en una articulacin, se resuelve, y luego se
desarrolla en otra articulacin. El dolor no empeora ni mejora por la actividad. Niega
rigidez matinal. Refiere historia de una erupcin en su hombro que apareci
inmediatamente despus de una de sus excursiones de cacera, y resolvi hace 6
semanas. El paciente trabaja como oficial de polica, y va de cacera 2 veces al mes.
9

Bebe 1 o 2 cervezas ocasionalmente. Su temperatura es de 37.8C y su pulso de 75 cpm.


Cul de los siguientes microorganismos es la causa ms probable de sus sntomas?
A) Borrelia burgdorferi
B) Borrelia recurrentis
C) Staphylococcus aureus
D) Treponema carateum
E) Treponema pallidum
25) Un nio de 4 aos con una hidrocefalia congnita es trado al cuarto de urgencias con
fiebre, dolor de cabeza, irritabilidad, fotofobia y vmitos de 2 das de evolucin. l fue
operado hace un ao para una derivacin ventrculo-peritoneal y sus vacunas estn al
da. Su temperatura es de 39.6 C. Al examen fsico, tiene rigidez nucal y signos de
Kernig y Brudzinski presentes. La piel que rodea el trayecto del catter de derivacin, est
eritematosa. Una puncin lumbar y posterior anlisis del LCR, muestra un conteo de
leucocitos de 40 000 x mm3 con 85% de neutrfilos, Glucosa en 48 mg/dl y protenas en
169 mg/dl. Cul de las siguientes opciones, es la causa ms probable de la condicin
actual del paciente?
A) Haemophilus influenzae
B) Neisseria meningitidis
C) Pseudomonas aeruginosa
D) Staphylococcus epidermidis
E) Streptococcus pneumoniae
26) Un hombre de 63 aos viene al mdico, despus de que un farmacutico le dice que
su medicacin es incorrecta. l tiene una historia de cirrosis heptica e hipertensin
portal, causada por abuso del alcohol. l nunca ha tenido historia de sangrado
gastrointestinal y recientemente se jubil de su trabajo como piloto comercial en una
aerolnea. l fuma un paquete de cigarrillos todos los das y actualmente, no toma alcohol.
La administracin conjunta de los siguientes frmacos, es probable que sea peligroso?
A) Lactulosa y potasio oral
B) Prednisona y salbutamol inhalado
C) Propranolol y dinitrato de isosorbide
D) Espironolactona e hidroclorotiazida
E) Espirolactona y potasio oral
27) Una mujer de 52 aos quien est siendo tratada con quimioterapia despus de una
ciruga por cncer de mama, es trada al cuarto de urgencias. Ella tom un antiemtico
ayer para las nuseas y fue trada por su esposo porque ha tenido un comportamiento
bizarro en las ltimas 24 horas, con flexibilidad crea en sus movimientos y mutismo. Ella
no tiene historia psiquitrica previa y no tiene otras condiciones mdicas, adicional al
cncer de mama. El examen fsico, los laboratorios y signos vitales, se encuentran todos
normales. Cul de las siguientes opciones, es la farmacoterapia ms apropiada para el
manejo?
A) Alprazolam
B) Benztropina
C) Haloperidol
D) Metilfenidato
E) cido valproico
10

28) Una mujer de 24 aos viene a su consultorio para su evaluacin, posterior a la visita
que ella hizo hace dos das al cuarto de urgencias por una reaccin alrgica que
consiste en una picazn en la garganta despus de comer helado, seguido de nuseas y
vmitos, una sensacin de rubor y urticaria. Este mismo tipo de problema le pas hace un
ao, despus de comerse un dulce de chocolate y una vez antes, en un restaurante de
comida tailandesa. Su amigo sugiri que esto puede ser una alergia al man, pero la
paciente no recuerda si ha comido man cuando le han ocurrido esas reacciones. En este
momento, todas las manifestaciones de la reaccin alrgica se han resuelto y ella luce
saludable y no tiene una historia significativa por enfermedades previas, su nico
medicamento actualmente, son anticonceptivos orales. Al examen fsico en el consultorio,
todo est normal. Cul de las siguientes opciones es el siguiente paso ms apropiado en
la evaluacin y el manejo de esta paciente?
A) Iniciar desensibilizacin (inyecciones de alrgeno) con extracto de man
B) Decirle a la paciente que haga un registro de los alimentos que consume hasta el
prximo episodio alrgico
C) Iniciar un desafo con man
D) Ordenar un prick test (prueba de pinchazo) en la piel o una prueba de
radioalergosorbencia para alergia al man
E) Iniciar terapia profilctica con bloqueadores H2 y difenhidramina
29) Masculino de 50 aos que acude por examen rutinario de mantenimiento de la salud.
Historia mdica sin complicaciones, no toma medicamentos. Ha fumado 2 paquetes de
cigarrillos diarios por dos aos, pero dejo de fumar hace 27 aos. Su hermano muro de
cncer pulmonar de clulas escamosas a los 56 aos. Presin arterial de 122/84 mmHg,
pulso de 74/min, Fr: 12/min. Examen fsico sin anormalidades. Est muy ansioso por la
muerte reciente de su hermano causado por el cncer. Cul es el mayor riesgo que puede
padecer el paciente de las siguientes malignidades?
A) Carcinoma colorectal
B) Carcinoma hepatocelular
C) Carcinoma pulmonar
D) Carcinoma pancretico
E) Carcinoma prosttico
30) Femenina de 47 aos que acude por numerosas quejas incluyendo dolor seo,
mialgia, constipacin y humor deprimido. Ella ha estado hospitalizada dos veces el ao
pasado por urolitiasis. Los estudios de laboratorios revelan hipercalcemia y niveles
elevados de hormona paratiroidea. La funcin renal esta normal, y un USG del cuello
revela masa paratiroidea. Cul de los siguientes anormalidades del laboratorio podran
esperarse en esta paciente?
A) Hipernatremia
B) Hiperfosfatemia
C) Hiponatremia
D) Hipofosfatemia
D) Fosfatasa alcalina baja
31) Un hombre de 48 aos acude al mdico por prdida de peso, fatiga y una erupcin de
1 ao de evolucin. Niega nauseas/vmito, disfagia, hematoquezia, melena, diarrea o
constipacin. Ha tenido tos productiva de esputo teido con sangre y tambin sangre en
la orina. No fuma. Recientemente visit un rea rural de India hace 2 semanas. Tiene
11

historial de asma bronquial de larga data. Su medicacin actual consiste en albuterol


inhalado para la falta de aire. Su temperatura es de 37.8C, presin arterial 122/68, pulso
de 89 cpm y frecuencia respiratoria de 15 cpm. Su saturacin de oxgeno es de 98% a
aire ambiente. El examen fsico muestra dolor a la percusin de los senos paranasales.
Las membranas mucosas estn secas. A la auscultacin se escuchan sibilancias
dispersas y sonidos respiratorios gruesos. Examen cardiaco con S1 y S2 normales, sin
soplos audibles. No hay sensibilidad a la palpacin abdominal. Se observa una erupcin
elevada, palpable y papular que no blanquea con la presin en la parte anterior de sus
piernas. La fuerza muscular es 3/5 en dorsiflexin del pie izquierdo y 2/5 para la extensin
de la rodilla derecha. No hay signo de Banbinski. Una TC de trax muestra engrosamiento
bronquial.
Laboratorios: glbulos blancos =12,100/mm3.
Neutrfilos 75%,
Linfocitos 8%,
Eosinfilos 10%
Hematocrito: 31%
Plaquetas: 490,000/mm3
Na+: 34 mEq/L,
K+: 5.3 mEq/L,
Cl-: 100 mEq/L
HCO3-: 20 mEq/L
Nitrgeno de urea: 35 mg/dl
Creatinina:2.2 mg/dl
Glucosa:103 mg/dl
AST: 25 U/L,
ALT=31 U/L,
C3: 90 mg/dl
C4: 20 mg/dl
Fosfatasa alcalina: 90 U/L
Anticuerpo Antinuclear: positivo 1:128
Anticuerpos Anti-citoplasma de neutrfilo (ANCA): positivo 1:256
PPD: No reactivo.
Urinlisis: cilindros hemticos Cul es el diagnstico ms probable?
A) Enfermedad de Churg-Strauss
B) Apergilosis diseminada
C) Tuberculosis diseminada
D) Carcinoma oculto
E) Granulomatosis de Wegener
32) Un hombre de 50 aos acude al mdico por un dolor cerca de sus orejas de 1 semana
de evolucin. Hace 3 das sus orejas se comenzaron a hinchar. Niega cualquier cambio
en la dieta o viajes recientes. Su historial mdico es irrelevante. Toma multivitaminas
diariamente. Su temperatura es de36.8C, presin arterial de 110/80 y pulso de 75 cpm.
Su saturacin de oxgeno es de 94% a aire ambiente. El examen fsico muestra
sensibilidad a la palpacin e hinchazn sobre las porciones cartilaginosas de ambas
orejas. Las reas no cartilaginosas no muestran anormalidad. Una biopsia de la oreja
muestra tejido de granulacin en el cartlago. Cul de las siguientes pruebas de funcin
pulmonar es ms probable que presente este paciente?
A) Disminucin de la difusin de monxido de carbono
B) Disminucin es flujo espiratorio
C) Incremento en FEV1
D) Reduccin de la capacidad residual funcional
E) Reduccin de la capacidad vital
33) Un albail de 27 aos llega a emergencias con su rodilla izquierda hinchada. El dolor
empez hace 36 horas y le ha limitado su capacidad para trabajar. Ha tenido fiebre y
escalofros desde las ltimas 24 horas. El examen fsico revela una temperatura de
38.9C y pulso de 104 cpm. El resto del examen fsico solo es importante para una rodilla
izquierda hinchada, eritematosa y con evidente efusin. El rango de movimiento se
encuentra limitado. La artrocentesis revela 90,000/mm3 leucocitos con 82% de neutrfilos.
12

Un frotis Gram revela muchos neutrfilos, pero no se visualizan microorganismos. La


microscopia de luz polarizada no revela cristales. Basado en la etiologa ms probable,
cul de las siguientes es el tratamiento inicial ms apropiado:
A) Vancomicina
B) Colchicina
C) Ceftriaxona
D) Nafcilina
E) Ceftriaxona+vancomicina
34) Una mujer de 74 aos acude al mdico por historia de 3 meses de dolor seo. El dolor
no se alivia con ibuprofeno. Tiene historia de hipertensin y enfermedad coronaria. Sus
medicamentos incluyen lisinopril, metoprolol y aspirina. Ha fumado 2 paquetes de
cigarrillos diarios por 30 aos. No consume bebidas alcohlicas. Su temperatura es de
36.8C, presin arterial 130/85 y pulso de 65 cpm. El examen fsico muestra dedos en
palillo de tambor (dedos hipocrticos) en ambas manos. A la auscultacin se escucha una
fase espiratoria prolongada y un galope S4. Hay sensibilidad a la palpacin de las
porciones distales de sus huesos. Radiografas de antebrazos y fmur revelan periostosis
bilateral. Cul de las siguientes opciones es la ms adecuada para el siguiente paso en
el manejo?
A) Biopsia de mdula sea
B) Radiografa de trax
C) Incrementar la dosis y frecuencia del ibuprofeno
D) Aspirado articular
E) No est indicado manejo adicional
35) Una mujer de 60 aos se queja de incremento de fatiga desde hace unos meses
atrs. Reporta que sus actividades diarias regulares le producen dolor y rigidez,
especialmente lavarse el cabello. Tambin ha tenido problemas para encontrar una
posicin cmoda para descansar. Ha perdido 12 libras de peso corporal en los ltimos 3
meses, pero antes de esto se senta saludable y no est bajo ninguna medicacin. El
examen fsico es normal. Cul de los siguientes es el diagnstico ms probable?
A) Depresin
B) Hipokalemia
C) Polimialgia reumtica
D) Polimiositis
E) Arteritis de la temporal
36) Una mujer de 26 aos acude al mdico por historia de 3 meses de dolor en las
articulaciones y en las manos. Refiere mnima mejora con ibuprofeno; el dolor ha estado
interfiriendo con su trabajo como arquitecta. Ella ha tenido una erupcin enrojecida
despus de hacer deportes en exteriores. Ha tenido dolor en la boca mientras cepilla sus
dientes. Su temperatura es de 37.4C, presin arterial de 130/90, pulso de 76 cpm. El
examen fsico muestra hinchazn en la articulacin interfalangica proximal del dedo ndice
de la mano izquierda. Tiene multiples lceras puntiformes en la mucosa oral.
Labs: Na+: 141 mmol/L
HCO3-: 23mmol/L
Leucos: 4,700/mm3
Eosinfilos:1%

K+: 5.0 mmol/L


Cl-: 97 mmol/L;
Hb: 12.5 mg/dl
Plaquetas: 20,000/mm3
Neutrfilos segmentados: 55%
Bandas: 3%;
Basfilos: 0.50%;
Linfocitos: 33%
Monocitos: 3%
13

ANA srico positivo y Anti-dsDNA positivo Cul de las siguientes opciones es la ms


probable como indicacin para el uso de esteroides en este paciente?
A) Anemia hemoltica
B) Dolor articular refractario a ibuprofeno
C) Trombocitopenia
D) ANA srico positivo
E) Pericarditis
37) Una mujer de 56 aos acude al mdico por un severo dolor de espalda y dolor de
cadera moderado desde hace 2 das. Los sntomas empezaron cuando intent levantar a
su gato. Ha tenido fatiga y malestar general desde hace 3 meses. Niega disfuncin
neurolgica o trauma. SU historial mdico es irrelevante. Su presin arterial es de 135/85
y pulso de 80 cpm. Al examen fsico presenta extrema sensibilidad a la palpacin en la
columna media lumbar e incapacidad para flexionar y extender la espalda debido al dolor.
Las radiografas de la columna lumbar y pelvis muestran una fractura por compresin de
la vrtebra L3; tambin hay varias reas de radiolucencias redondas en su pelvis y fmur
proximal. Los laboratorios muestran lo siguiente:
Hb: 11g/dl
VCM=85fl;
Glbulos blancos: 8,000/mm3;
Plaquetas=200,000/mm3; Protenas totales: 9.8 mg/dl;
Albmina: 3.0mg/dl;
Ca2+:12.0mg/dl;
Fosfatasa alcalina: 65U/L
VES: 35mm/hr
Cul de los siguientes es el diagnstico ms probable?
A) Herniacin de disco lumbar
B) Cncer metastsico a hueso
C) Mieloma Mltiple
D) Osteoporosis
E) Enfermedad de Paget sea
38) Un hombre de 56 aos es admitido al hospital porque tiene dificultad respiratoria. Se
le coloca un tubo endotraqueal para darle ventilacin mecnica con un volumen corriente
de 900 cc con una frecuencia respiratoria de 12 por minuto y una FIO2 de 50%. La
presin positiva al final de la espiracin es de 10 cmH2O. Sus medicamentos actuales
incluyen heparina subcutnea y aspirina. Dos das despus, l desarrolla taquicardia y
tiene una presin arterial de 110/70 mmHg. Al examen cardaco se encuentran
contracciones prematuras mltiples. Una gasometra muestra una pO2 de 40 mmHg. Cul
de las siguientes opciones es la causa ms probable de la condicin del paciente
A) Secreciones bronquiales
B) Arritmia cardaca
C) Infarto al miocardio
D) Neumotrax
E) Tromboembolismo pulmonar
39) Un estudiante de medicina de 22 aos viene a la clnica quejndose de debilidad
severa, dolor de garganta y fiebre de 38.9 C desde hace 4 das, adicional un exantema
generalizado de color rosado durante los ltimos dos das. Ha tenido buena salud antes
de este episodio y despus de obtener una historia clnica y de realizar el examen fsico,
se ha constatado que el paciente no ha tenido contacto con personas enfermas, l tiene
una relacin monogmica heterosexual desde hace un ao y no ha viajado en el ltimo
ao. El niega el uso de drogas ilcitas y admite que se ha automedicado con amoxicilina
14

para su faringitis estreptoccica. l dice que yo ahora he desarrollado fiebre escarlatina.


Doctor, aydeme. El aparenta estar plido y exhausto. El examen fsico muestra una
linfadenopata difusa e inflamada y sus amgdalas estn eritematosas. Se le palpa la
punta del bazo y el borde heptico y tienen un ligero dolor a la palpacin. Un hemograma
completo se le realiza en el momento y revela lo siguiente:
Hematocrito: 42%
Hemoglobina: 14 g/dl
Leucocitos: 4500 x mm3
Neutrfilos: 35%
Eosinfilos: 2%
Monocitos: 3%

Plaquetas: 140,000 x mm3


Linfocitos: 60%

Se le realiz un frotis sanguneo en el que se encontraron >10% de linfocitos con


abundante citoplasma. Cul de las siguientes opciones, es el siguiente paso para
confirmar el diagnstico?
A) Aminotransferasas hepticas y bilirrubina
B) Prueba de anticuerpos heterfilos
C) Prueba de IgG para el antgeno de la cpside del virus de Epstein Barr
D) IgM para citomegalovirus
E) IgM para Toxoplasma
40) Un hombre de 24 aos viene al mdico porque tiene una historia, desde hace una
semana de fiebre, tos productiva con sangre, sibilancias y dificultad para respirar. l ha
desarrollado un exantema pruriginoso en el pecho y el abdomen desde ayer. Es granjero
en una zona rural del sureste de los Estados Unidos. Tiene una frecuencia respiratoria de
30 por minuto y a la auscultacin presenta sibilancias, adicional presenta un exantema
urticariforme en el tronco. Al examen abdominal tiene una distensin leve y malestar
difuso a la palpacin. Se le realiza una radiografa de trax y muestra varios infiltrados en
forma de parche, bilaterales, pero tiene ms en el campo pulmonar derecho. En los
laboratorios se evidencian:
Hemoglobina: 14.3 g/dl
Leucos: 5 400 x mm3
Eosinfilos: 30%
Neutrfilos segmentados: 35%
Linfocitos: 30%
Monocitos: 5%
Cul de los siguientes, es la farmacoterapia ms apropiada para el tratamiento?
A) Ivermectina
B) Levamisol
C) Mebendazol
D) Pamoato de pirantel
E) Tialbendazol
41) femenina de 39 aos primigrvida de 40 semanas de gestacin acude a sala de parto
con contracciones, sus controles prenatales fueron sin complicaciones. Ella tiene
sntomas de hipotiroidismo para la cual toma levotiroxina. No tiene alergias a
medicamentos conocidos. Su laboratorio prenatal muestran:
Tipo de sangre: A negativo
Antgeno de superficie de hepatitis B: Negativo
Rubeola: Inmune
Reagina rpida en plasma: No reactivo
HIV: Negativo
Prueba de carga de glucosa: 100mg/dL

15

La paciente da a luz a una nia de 6 libras, 14 onzas con puntaje de APGAR 9/9. El tipo
sanguneo de la nia es A positivo. El plan de la paciente es alimentar al infante con
lactancia materna exclusiva. Cul de los siguientes es el paso ms apropiado en el
manejo?
A) Administrar inmunoglobuina antihepatitis B y vacuna de la Hepatitis B
B) Administrar RhoGAM
C) Administrar inmunizacin para rubeola
D) Empezar metroclopramida
E) Comenzar pldoras de anticonceptivos orales
42) Una bebe de 8 das de edad es trada al mdico por sus padres debido a una
decoloracin cutnea amarillenta. Ella naci a las 39 semanas de gestacin y ha sido
alimentado con pecho desde entonces. Examen fsico muestra ictericia significativa del
tronco y cara. Sin ictericia escleral. Por otra parte se encuentra saludable. Una revisin de
su historial muestra que tanto su madre y ella son Rh-positivo. Cul de los siguientes es
la causa ms probable de la condicin de la infante?
A) Ictericia por leche materna
B) Deficiencia de glucosa 6 fosfato deshidrogenasa
C) Ictericia por falla en alimentacin de leche materna
D) Ictericia fisiolgica
E) Incompatibilidad por Rh
43) Femenina de 38 aos grava 3 para 2 a la dcima semana de gestacin que acude
para control prenatal. Se siente muy bien solo con nuseas ocasionales. Sus historias
obsttricas fueron de dos partos vaginales espontneos a trmino sin complicaciones de
embarazo. Tiene como antecedentes medico significativo la hipertensin y ha estado en
medicacin de vez en cuando, sin embargo no recuerda el nombre del medicamento. Ella
nunca ha tenido una ciruga y no ha tenido alergias a medicamentos. Su Presin arterial
inicial es de 144/92 mmHg. Al Examen fsico fue normal para una mujer en su 10 semana
de gestacin. Se le repite la Presin arterial y es de 140/90mmHg. 2 semanas despus
ella regresa para toma de Presin arterial la cual tuvo en 142/92mmHg. Cul de los
siguientes es el prximo paso ms apropiado en el manejo?
A) Continuar con seguimiento de la Presin arterial
B) Empezar con alfa metildopa
C) Empezar con atenolol
D) Empezar con captopril
E) Empezar con labetalol
44) Un hombre de 66 aos acude al mdico por debilidad progresiva desde hace 3
meses. El paciente ya no es capaz de subir escaleras. El paciente presenta una erupcin
en la parte superior del trax y en la cara. Niega hormigueo, adormecimiento, cefalea,
dificultad para tratar o incontinencia. No toma medicamentos. Su temperatura es de
37.2C, presin arterial de 135/80 y pulso de 80 cpm. El examen fsico muestra lesiones
eritemato-escamosas en las reas ciliares, laterales al tabique nasal y prpados, en
ambos lados de la cara. Se evidencia debilidad bilateral de la musculatura proximal;
reflejos osteotendinosos disminuidos. Laboratorios se muestran a continuacin

16

Na+: 138 mEq/L


Cl-: 102 mEq/L
K+: 4.4 mEq/L
HCO3-: 25mEq/L;
Creatina Kinasa: 30,000 ng/Ml
Anticuerpo antinuclear: Positivo
Cul de las siguientes opciones es el siguiente paso ms apropiado en el manejo?
A) Prednisona
B) Electromiograma
C) Immunoglobulina IV
D) Biopsia de msculo
E) Plasmaferesis
45) Una mujer de 23 aos acude al mdico por dolor en el lado derecho del trax que
empeora a la inspiracin. Refiere que ha tenido una erupcin pruriginosa en la cara e
hinchazn dolorosa en sus dedos de las manos. No toma medicamentos. Su temperatura
es de 37C, presin arterial de 160/90, pulso de 98 cpm y frecuencia respiratoria de 15
cpm. El examen fsico muestra lesiones papuloeritematosas en ambas mejillas y el dorso
de la nariz. La paciente presenta hinchazn de las articulaciones interfalngicas
proximales y metacarpofalngicas bilateralmente. Los ruidos respiratorios estn
disminuidos en el campo pulmonar inferior derecho. La hemoglobina es de 8g/dL. Cul
de las siguientes pruebas diagnsticas es ms probable que se encuentra anormal?
A) Anticuerpos anti-centromero
B) Anticuerpos anti-DNA de doble cadena
C) Anticuerpos anti-histonas
D) Anticuerpos anti-Jo-1
E) Anticuerpos anti-nucleares
46) Una mujer de 32 aos viene al mdico para una evaluacin de rutina. Ella dice que ha
tenido palpitaciones episdicas pero niega que ha tenido dolor en el pecho, falta de
aliento o otros sntomas significativos. No ha tenido historia de enfermedades relevantes y
no toma medicamentos. Su temperatura es de 36.4 C, Presin arterial de 120/70 mmHg,
pulso en 72 x minuto, respiratorio 16x minuto, Peso de 65 kg, altura de 170 cm y IMC de
22.5 kg/m2. Al examen fsico no se encuentran anormalidades y los laboratorios estn
normales. Un ECG muestra contracciones atriales prematuras ocasionales y un
ecocardiograma, muestra una FE normal y no presenta anomalas valvulares. Cul de
las siguientes opciones, es el mejor siguiente paso en el cuidado de la paciente?
A) Digoxina
B) Diltiazem
C) Tranquilizar a la paciente
D) Flecainida
E) Metoprolol
47) Un hombre de 50 aos que tiene una historia de abuso del alcohol durante los ltimos
20 aos, es trado al hospital, quejndose de fiebre, mareos y tos. Niega otros
antecedentes. Tiene tos productiva con esputo de aspecto marrn y niega otros sntomas,
no toma medicamentos y es alrgico a las sulfas. Tiene una temperatura de 37.5 C,
Presin arterial de 110/70 mmHg, pulso de 91 cpm y respiracin de 14 rpm Al examen
fsico se evidencia crpitos en el lbulo pulmonar superior izquierdo y al examinar el
corazn, tiene ritmo y frecuencia regular. Una radiografa de trax muestra que tiene un
infiltrado en el lbulo superior del pulmn izquierdo. Cul sera el manejo farmacolgico
ms apropiado?
17

A) Azitromicina
B) Cefuroxima y azitromicina
C) Clindamicina
D) Eritromicina y levofloxacina
E) Vancomicina
48) Una mujer de 34 aos HIV positiva viene al mdico con su pareja porque tiene una
historia de malestar y artralgias desde hace 4 das. Su pareja dice que la paciente ha
tenido episodios de confusin y sensacin de hormigueo en sus brazos y piernas desde
hace 2 horas, su pareja est preocupada porque la paciente us herona hace 3 das y
todava est sangrando en el sitio donde se inyect. Sus medicamentos son zidovudina,
didanosina, indinavir y trimetoprim con sulfametoxazol. La paciente est orientada en
persona, pero no en tiempo y lugar. Su temperatura es de 39.1, PA 150/90 mmHg, pulso
en 110 x y la FR en 20 x. Al examen fsico se evidencian escleras ictricas y tiene
petequias no palpables en el pecho y en las extremidades, sus pulmones estn claros a la
auscultacin y a la palpacin abdominal, tiene sensibilidad difusa y leve a la palpacin. La
punta del bazo se le palpa a 4 cm por debajo del reborde costal. Los laboratorios
muestran lo siguiente:
Hemoglobina: 7.1 g/dl
Plaquetas: 50 000 x mm3
CD4: 230 x uL
Leucos: 4 600 x mm3
Tiempo de sangra: 11 minutos (N: 2-7 min)
Frotis de sangre perifrica: Glbulos rojos fragmentados
Protenas en orina: 500 mg/dl
Glbulos rojos en orina: 50 x campo
Cul de los siguientes hallazgos seran los ms probables en esta paciente?
A) Niveles de creatinina normales
B) Niveles de bilirrubina indirecta normales
C) Niveles de lactato deshidrogenasa normales
D) Tiempos de protrombina y de tromboplastina parcial activada normales
E) Conteo de reticulocitos normales
49) Femenina de 60 aos con historia de osteoartritis acude al mdico debido a un dolor
en su rodilla izquierda. Su dolor no se alivia con 2,2 mg de ibuprofeno por da. Ella no
presenta dolor abdominal ni sangre en heces. Ella ha ganado 4,5 Kg (10 lb) de peso
durante el ltimo ao. Ella pesa 90,7 Kg y talla 160 cm (63in). Su temperatura es 37C
(98,6F), Presin arterial de 125/75 mmHg, pulso de 70/min. Tiene una efusin articular
de tamao mediano en la rodilla izquierda. Hay una sensibilidad difusa a la palpacin,
crpitos e inestabilidad articular leve tanto medial como lateral. Cul de los siguientes es
el prximo paso ms apropiado en el manejo?
A) Adicionar acetaminofn a su rgimen
B) Incrementar dosis de ibuprofeno
C) Inyeccin intraarticular de esteroides, cada 2 semanas por 6 meses
D) Reemplazo de rodilla
E) Prdida de peso ms ejercicios de resistencia
50) Masculino de 66 aos es trado a sala de emergencias aquejndose de fatiga, nausea
vmito y pobre apetito por una semana. Tiene historia de HTA de larga data, bien
controlada con lisinoprol, 40mg/da y nifedipina, 90mg/da, diabtico y sufre de
insuficiencia renal crnica. Ha comenzado con dilisis hace 1 ao pero no ha sido capaz
de ir en las ltimas dos semanas debido a falta de transporte. Su temperatura es de 37C
18

(98,6F), presin arterial de 186/82 mmHg, pulso de 90/min y frecuencia respiratoria de


20/min al examen fsico muestra distensin venosa yugular en 5cm y crpitos bibasales
en los pulmones, y al examen cardiaco revela un S3, sin frote pericrdico, edema +1 en
extremidades bajas. Estudios de laboratorios revelan:
Hb: 8,8g/dL
Cuenta leucocitaria: 8,000/mm3
Na+: 139 mEq/L
Cl-: 100 mEq/L
HCO3-: 14 mEq/L
Nitrgeno de urea: 156 mEq/L
Glucosa
112 mg/dL

Plaquetas: 186,000/mm3
K+: 7,9 mEq/L
Creatinina: 7,2 mg/dL

EKG muestra un ritmo sinusal con elevacin de la onda T y ensanchamiento del complejo
QRS, el paciente recibe una ampolla de gluconato de calcio, una ampolla de 50% de
dextrosa con 10 unidades de insulina regular, y una ampolla de bicarbonato de sodio.
Cul de los siguientes es el prximo paso ms apropiado en el manejo del potasio
anormal en el paciente?
A) Administrar Kayexalate (polistireno de sodio)
B) Hemodilisis de emergencia
C) Administrar 120 mg de furosemida IV
D) Enviar al paciente a casa y organizar transporte para paciente externo de dilisis
E) Detener lisinopril y monitorear niveles de potasio
51) Una mujer de 42 aos se presenta con el mdico porque tiene dolor epigstrico. Se ha
dado cuenta de este sntoma desde las ltimas semana, es espordico y est asociado
con las comidas. Ella niega alguna asociacin con un tipo de comida y su historia mdica
no tiene complicaciones. A pesar de que ella se queja de pirosis, toma anticidos para
esto. Niega prdida de peso, hematemesis o melena y al examen fsico, tiene temperatura
de 37C y la presin arterial en 110/70, la cabeza, cuello, trax, cardiovascular estn
normales y al examen abdominal, tiene sensibilidad a la palpacin del epigastrio, no tiene
hepatoesplenomegalia ni tampoco tiene signo de Murphy, por otra parte, el abdomen est
blando, sin ascitis. El resto del examen est normal- Se le realiza una Serie
esofagogastroduodenal donde se evidencia una lcera redonda con bordes lisos en el
bulbo duodenal. Cul de las siguientes opciones, es el siguiente paso ms apropiado en
el cuidado de esta paciente?
A) Biopsia de la lcera
B) Monitoreo del pH esofgico
C) Pantoprazol oral
D) Ranitidina oral
E) Prueba de aliento de ureasa.
52) Un hombre de 24 aos es hospitalizado para quimioterapia de induccin, previo a un
trasplante de mdula sea, el tiene una historia de 5 meses de leucemia mieloblstica
aguda y recibe altas docis de busulfn y ciclofosfamida. Un da despus, desarroll
nuseas y vmitos y no tolera alimentos, ni bebidas ni medicamentos. Su temperatura es
de 37.1 C. Cul de las siguientes opciones es el tratamiento farmacolgico ms
apropiado para este paciente?
A) Dopamina
B) Eritromicina
C) Ondansetrn

19

D) Metoclopramida
E) Omeprazol
53) Una mujer de 72 aos acude al mdico con dolor en la mitad de la espalda desde
hace 3 das. Niega trauma en el rea. El examen fsico muestra sensibilidad a la
palpacin del punto correspondiente a la vrtebra T8. Los laboratorios obtenidos son:
Hemoglobina en 13g/dl.
Sodio: 137 mEq/L;
Potasio: 4.3mEq/L;
Bicarbonato: 25 mEq/L
Nitrgeno de urea; 21 mg/dl
Calcio; 9.0 mg/dl;
Fosfatasa Alcalina: 50 U/L

Cloro: 101 mEq/L;


Creatinina: 0.7 mg/dl
Albmina=3.6 g/dl

Una radiografa de la columna muestra una fractura por compresin del cuerpo vertebral
de T8. La Absorsiometraa de Energa Dual de Rayos X (DEXA) muestra un T-score de
densidad mineral sea de -2.7. Cul de los siguientes es el diagnstico ms probable?
A) Osteitis deformante
B) Raquitismo
C) Osteomalacia
D) Osteopenia
E) Osteoporosis
54) Un hombre de 65 aos llega al departamento de emergencias por una historia de 3
meses de falta aire progresiva. Niega dolor en el pecho y fiebre. Hace 5 aos tuvo
mieloma mltiple, para el cual fue tratado. No toma medicamentos y no tiene alergias.
Bebe una cerveza en la cena diariamente. Su temperatura es de 37.2C, presin arterial
110/70, pulso de 105 cpm y frecuencia respiratoria de 22 cpm. Hay distensin venosa
yugular hasta el ngulo de la mandbula. Los pulmones estn claros a la auscultacin. El
examen cardiaco revela sonidos cardiacos distantes. Se ausculta S3 y S4. Hay
hepatomegalia y edema pedio 2+, Entre sus laboratorios se observan: Hierro srico en
100 ug/dL; Capacidad total de unin al hierro en 350 ug/dl y la Ferritina srica en 100
ng/mL. El EKG muestra disminucin de la amplitud de los complejos QRS. Una
radiografa simple de trax no muestra anormalidades. El ecocardiograma muestra un
miocardio de apariencia moteada. Cul de los siguientes es el diagnstico ms
probable?
A) Cardiomiopata alcohlica
B) Amiloidosis
C) Hemocromatosis
D) Cardiomiopata isqumica
E) Sarcoidosis
55) Una mujer de 38 aos acude al mdico por una erupcin en sus piernas desde hace 1
semana. Niega fiebre o prurito. Su historial mdico es irrelevante y no toma
medicamentos. Su temperatura corporal es de 36.9C, presin arterial de 125/85 y pulso
de 75 cpm. La saturacin de oxgeno se encuentra en 98% a aire ambiente. El examen
fsico muestra ppulas eritematosas elevadas, sensibles a la palpacin, sobre sus caras
anteriores de las piernas. El bazo es palpable 3c, por debajo del margen costal izquierdo.
La radiografa de trax simple muestra infiltrados hiliares bilaterales. Un espcimen de
biopsia de ndulo linftico hiliar derecho, muestra granulomas no caseosos. Cul de los
siguientes hallazgos se asocia con mejor pronstico para este paciente?

20

A) Eritema nudoso
B) Bloqueo cardiaco
C) Consolidado pulmonar con ndulos linfticos hiliares
D) Lupus pernio
E) Compromiso ocular
56) Un hombre de 63 aos llega a urgencias con dolor en su pierna izquierda luego de
resbalarse en el piso de su cocina. El paciente dice haber escuchado un chasquido al
impactarse contra el suelo. No ha tenido prdida de peso ni de apetito. El paciente tiene
historial de diabetes e hipertensin. Su medicacin actual consiste en metformina y
lisinopril. Su temperatura es de 36.9C y su presin arterial es de 130/80. El examen fsico
muestra hipoacusia bilateral. Se ausculta un soplo meso-sistlico en crescendodecrescendo de intensidad 2/6. El paciente presenta extrema sensibilidad a la palpacin
del muslo izquierdo, y moderada sensibilidad a la palpacin en el muslo no lastimado. Una
radiografa simple del muslo izquierdo muestra una fractura en el tercio medio de la
difisis del fmur y un contorno cortical expandido y deformado. Una radiografa del muslo
derecho muestra encorvamiento del fmur. Un centelleo seo muestra reas focales de
intensa captacin del medio de contraste en ambos muslos.
Laboratorios:
Na+: 140 mEq/L
K+: 4.8 mEq/L
Cl-: 105 mEq/L
HCO3-: 24 mEq/L
Creatinina: 1.0 mg/dl
Fosfato: 2.9 mg/dl
AST: 32 UI/L;
ALT: 25 UI/L
GGT: 15 UI/L
VES: 18 mm/hr
Nitrgeno de urea: 13 mg/dl;
Adems de referirlo a ciruga ortopdica, Cul de las
siguientes es la farmacoterapia ms apropiada?
A) Alendronato
B) Trasplante autlogo de mdula sea
C) Suplementacin con calcio
D) Doxorrubicina
E) Altas dosis de corticosteroides
57) Una mujer de 28 aos G1P0 con 15 semanas de embarazo, viene para su consulta
prenatal de rutina en noviembre. Ella no ha tenido nuseas ni vmitos y a la edad de 15
aos, fue operada por una fractura expuesta, el resto de la historia clnica es irrelevante.
Ha recibido todas las vacunas de la infancia, que se le han ofrecido y su ltima
vacunacin fue a la edad de 16 aos. Toma habitualmente acetaminofn para cefaleas
espordicas, no fuma, no consume alcohol ni drogas ilcitas. Cul de las siguientes
opciones, es la vacuna ms apropiada que se le puede administrar a la paciente en esta
consulta?
A) Vacunas de la Hepatitis A y hepatitis B
B) Vacuna de la hepatitis B y Toxoide tetnico Td
C) Vacuna de la influenza inactivada, MMR y varicela
D) Vacuna de la influenza inactivada y vacuna del neumococo polisacrida
E) Vacuna de la influenza inactivada y toxoide tetnico Td
58) Un nio de 9 aos, es trado al cuarto de urgencias por su madre porque l dijo que
su corazn va demasiado rpido y siente que le pesa Esto le inici hace 3 horas y
nunca le haba sucedido esto, de hecho, siempre ha sido un nio saludable, a excepcin
de enfermedades comunes de la infancia y no ha visto al doctor en los ltimos 2 aos. Al
auscultarlo, se le encuentra una Frecuencia cardiaca de 200 x min. Se le convierte
fcilmente a ritmo sinusal despus de que su cara es sumergida en solucin salina helada
21

por unos cuantos segundos. En este momento, al examen se le encuentra un soplo


holosistlico de grado 3, sobre el lado izquierdo, junto con un ritmo de galope que se le
escucha en el borde esternal izquierdo. Se le realiza un ECG y tiene frecuencia cardaca
normal, pero muestra un intervalo PR corto con una elevacin lenta del QRS. Cul de las
siguientes opciones, es el diagnstico ms probable?
A) Defecto septal atrial
B) Anormalidad de Ebstein
C) Estenosis pulmonar
D) Tetraloga de Fallot
E) Atresia tricuspdea
59) Un hombre de 49 aos es evaluado en el cuarto de urgencia por una cada reciente y
un trauma en la cabeza. La tomografa computada es normal y sus lesiones son
solamente una pequea laceracin arriba del ojo derecho. Su historia mdica es
significante por una gastritis crnica y osteoartritis, por la cual espordicamente toma
acetaminofn. Sus laboratorios muestran:
Glbulos rojos 4.1 x 106/ml
Hcto: 39%
Hb: 12 g/dL
Glbulos blancos: 7.500/ml
VCM: 110/mm3
Plaquetas: 140,000/mm3
Frotis de sangre perifrica: muestra neutrfilos hipersegmentados
Anticuerpos Anti -IF: negativo
Fosfatasa Alcalina: 50 U/L
LDH: 15 U/L
AST: 150 U/L
Bilirrubina Total: 1 mg/dL Los hallazgos son consistentes con cul de los siguientes
diagnsticos?
A) Abuso crnico de alcohol
B) Gastritis inducida por drogas
C) Anemia Perniciosa
D) Enfermedad lcero pptica
E) Hepatitis viral
60) Masculino de 32 aos sin historia de enfermedad mental tiene "problemas
ocupacionales". El siente que no avanza profesionalmente en su trabajo como un analista
financiero, sin embargo, l se desempea por encima de las expectativas en el trabajo
que realiza en privado. Refiere que "se paraliza del miedo" cuando da una presentacin
en su trabajo. Tambin no puede concentrarse y desarrolla palpitaciones y temblores
cuando se encuentra alrededor de extraos en las fiestas de su trabajo. Cul de los
siguientes tratamientos es el ms apropiado para prevenir los sntomas y mejorar su
desempeo?
A) Benzodiacepinas
B) Buspirona
C) Clomipramina
D) Psicoterapia grupal con sobrevivientes
E) Propanolol
61) Un hombre de 32 aos acude al mdico por dolor de espalda de 4 das de evolucin,
que apareci sbitamente durante un estudio diagnstico. El paciente dice que el dolor se
encuentra cerca de la columna al lado derecho. El paciente recientemente comenz a
levantar peso. Niega fiebre, rigidez matinal, adormecimiento u hormigueo en las
extremidades, debilidad o incontinencia. Refiere beber vino ocasionalmente y no utiliza
22

drogas ilcitas. Su temperatura es de 36.8C y pulso de 72 cpm. Al examen fsico no se


evidencian deformidades en la columna. El rango de movimiento est limitado por el dolor.
No hay puntos de sensibilidad a la palpacin en el rea afectada. La fuerza muscular es
4/5 bilateralmente en extremidades superiores e inferiores, reflejos osteotendinosos
normales. No hay sensibilidad a la palpacin en los msculos de la pierna.
Cul de los siguientes es el diagnstico ms probable?
A) Espondilitis anquilosante
B) Herniacin de disco intervertebral
C) Bursitis interespinosa lumbar
D) Espasmo de msculo lumbosacral
E) Lesin de articulacin sacroiliaca
62) Un hombre de 65 aos con historial de hipertensin y osteoartritis acude al mdico por
debilidad progresiva de 3 aos de evolucin. Tiene dificultad para levantarse de una silla,
calambres moderados en sus muslos y dificultad para tragar. Niega enfermedad reciente.
La fuerza muscular es de 4/5 bilateralmente en la musculatura proximal de miembros
inferiores. Los reflejos patelares estn disminuidos bilateralmente.
Laboratorios:
Hb: 12.9 g/dl
Hematocrito; 44%
Plaquetas: 255, 000/mm3
Protena C-Reactiva: 1.2 mg/dl
Actividad de creatina kinasa: 300 U/L

Leucos: 8,300/mm3

Los resultados de cul de las siguientes pruebas diagnsticas es ms probable que


pueda proveer el diagnstico definitivo en este paciente?
A) Anticuerpos anti-nucleares (ANA)
B) Electromiografia (EMG)
C) Pruebas de funcin heptica
D) RMN de la columna vertebral y ambas extremidades inferiores
E) Biopsia de msculo
63) Un joven de 15 aos es llevado al mdico por historia de hormigueo episdico en sus
manos, seguidos de decoloracin azul y luego roja; desde hace 3 meses. La totalidad del
episodio dura aproximadamente 5 minutos y usualmente ocurre cuando tiene frio. Tiene
cefaleas ocasionales. Niega fiebre, prdida de peso, debilidad, erupciones, artritis y falta
de aire. Tiene historia de asma inducida por ejercicio, tratada con albuterol. Su padre tiene
historia de hipertensin. El joven juega rugby para el equipo de la escuela. Niega fumar,
beber alcohol y usar sustancias ilcitas. Su temperatura es de 37.2C, presin arterial de
110/70, pulso de 70 cpm y frecuencia respiratoria de 12 cpm. Su saturacin de oxgeno es
de 99% a aire ambiente. El examen fsico es normal Cul de las siguientes opciones es
el siguiente paso ms apropiado en el manejo?
A) Solicitar niveles de anticuerpos anti-nucleares (ANA)
B) Solicitar niveles de complemento C3 y C4
C) Microscopa del lecho ungueal
D) Observacin
E) Solicitar niveles de Factor Reumatoideo

23

64) Una mujer de 60 aos con historia de diabetes mellitus tipo2 desde hace 30 aos,
acude al mdico por un pie hinchado. Refiere haber tenido hinchazn, eritema y dolor
moderado en su pie derecho desde hace 3 meses. Ha tenido dificultad para caminar
debido a los sntomas. Tiene historia de hipertensin desde hace 12 aos. Sus
medicamentos actuales son metformina, insulina, lisinopril e hidroclorotiazida. Niega
fumar, consumo de alcohol y de drogas ilcitas. Su temperatura es de 36.8C, pulso de 86
cpm y saturacin de oxgeno de 94% a aire ambiente. Se auscultan crpitos bilaterales
bibasales y un S4. Se auscultan soplos carotdeos bilaterales. El pie derecho est
edematoso, indurado y caliente al tacto. Hay un dolor moderado a la manipulacin del pie
y disminucin de la sensacin de pinchazo y estmulo vibratorio bilateralmente debajo del
nivel de la rodilla. HbA1C de 9.1%, Glbulos blancos en 5,500/mm3. Cul de los
siguientes es el diagnstico ms probable?
A) Enfermedad por depsito de pirofosfato de calcio
B) Celulitis
C) Articulacin de Charcot
D) Gota
E) Artritis sptica
65) Una mujer de 32 aos con hipertensin, acude al consultorio por dificultad para tragar,
la paciente dice que ha tenido disfagia para lquidos y slidos desde hace 4 meses y ha
perdido aproximadamente 10 libras en ese mismo tiempo- Ella es saludable, a excepcin
de estos episodios de disfagia, que se alivian al tomar agua. Al examen, est
moderadamente obesa pero no est intranquila. Su temperatura es de 37C, Presin
arterial de 137/84 y pulso en 72 x minuto El resto del examen fsico est normal. Se le
ordena un estudio de bario que muestra un encintamiento liso, con forma de punta en el
esfago distal y dilatacin del esfago proximal. Se le ordena una endoscopia superiorCul de las siguientes opciones, es la explicacin ms probable de los sntomas?
A) Acalasia
B) Espasmo esofgico difuso
C) Cncer esofgico
D) Anillo de Schatzki
E) Esclerodermia
66) Una mujer caucsica de 36 aos con historia de hipertensin y diabetes mellitus
controlada con dieta, llega al cuarto de urgencias con dolor abdominal de dos das de
evolucin. El dolor se ubica en el epigastrio y se irradia a la espalda, tiene algo de
nuseas pero no vmitos y niega diarrea. En los ltimos dos aos, ha tenido 3 episodios
de dolor sordo en el cuadrante superior que se exacerbaba al consumir comida grasosa y
esto se resolva espontneamente. Toma lisinopril para la presin y niega episodios
anteriores de dolor abdominal y no fuma. Se toma dos cervezas cada fin de semana y la
historia familiar no contribuye. Al examen fsico, se ve a una paciente joven con
intranquilidad moderada, temperatura en 37.2C, Presin arterial en 110/70 mmHg, pulso
en 110 x min y respiratorio en 16 x min. Los pulmones estn claros a la auscultacin y el
examen cardiovascular no muestra soplos, galope ni frote. Al examen abdominal, hay
sensibilidad a la palpacin en la regin epigstrica sin defensa ni rebote o rigidez. Los
laboratorios muestran lo siguiente:
Hemoglobina: 12.6 g/dl
Glbulos blancos: 13,000 x mm3 con 18% de bandas
Plaquetas: 370,000 x mm3
Amilasa: 1900 U/l
Lipasa: 1600 U/l
Calcio: 8.2 mg/dl
Triglicridos: 800 mg/dL (N: 35-160 mg/dl)
24

AST: 80 U/L
ALT: 150 U/L
Cul de las siguientes opciones, es el diagnstico ms probable?
A) Colecistitis aguda
B) Pancreatitis alcohlica
C) Pancreatitis inducida por frmacos
D) Pancreatitis biliar
E) Pancreatitis por hipertrigliceridemia
67) Un hombre de 52 aos, con hepatitis C y cirrosis, es trado al cuarto de urgencias por
su esposa porque tiene 3 das de tener dolor abdominal y un da de confusin. El
paciente se le inici furosemida hace una semana por tener edema en las piernas y
abdominal. Los otros medicamentos, incluyen propranolol para las vrices esofgicas.
Aparenta estar somnoliento pero responde al llamado, la temperatura est en 37C, PA en
110/60 mmHg, pulso en 100 por min y respiratorio en 20 x min. Al examen neurolgico,
tiene asterixis; tiene el abdomen distendido con sensibilidad a la palpacin difusa, no tiene
defensa ni rebote. Tiene matidez cambiante y edema de tobillo 1+, tiene los niveles de
amonio en plasma elevados. Se le hospitaliza y se le administra lactulosa. Cul de las
siguientes opciones, es el siguiente paso ms apropiado para el manejo?
A) Tomografa de cabeza
B) Ecocardiograma
C) Lorazepam para la confusin del paciente
D) Espironolactona
E) Paracentesis guiada por ultrasonido
68) Un hombre negro de 46 aos con una historia de trastorno esquizoafectivo es
hospitalizado por psicosis aguda. Presenta alucinaciones auditivas, paranoia, depresin
con ideacin suicida, anhedonia, afecto aplanado, apetito aumentado y una ganancia de
peso de 10 libras en el ltimo mes y aumento en la somnolencia. El paciente es
estabilizado con haloperidol Cul de las siguientes opciones es el mejor antipsictico
atpico para recetarle al paciente?
A) Clozapina
B) Olanzapina
C) Quetiapina
D) Risperidona
E) Ziprasidona
69) Un hombre de 34 aos viene al mdico porque tiene dificultad para tragar, progresiva,
desde hace 4 meses. l ha tenido sensacin de que se atora despus de comer y
tambin episodios frecuentes de quemazn en el pecho. En los ltimos 10 aos, ha
presentado dificultad para tragar alimentos blandos y tiene historia de enfermedad por
reflujo gastroesofgico desde hace 5 aos. Su medicacin habitual incluye ranitidina,
hidrxido de magnesio y omeprazol. Fum 1 paquete de cigarrillos diarios durante 15
aos y no consume alcohol. Al examen fsico no se muestran anormalidades. Cul de
las siguientes opciones, es la explicacin ms probable de los sntomas del paciente?
A) Espasmo esofgico difuso
B) Carcinoma escamoso esofgico
C) Membrana esofgica inferior

25

D) Constriccin pptico-esofgica
E) Esclerodermia
70) Una mujer de 64 aos, viene al mdico porque tiene la conjuntiva seca, fotofobia y
sensacin de cuerpo extrao en sus ojos. Estos sntomas son agravados por un esfuerzo
visual prolongado, como leer y ver televisin. SU historia mdica es relevante para
hipertensin, enfermedad coronaria y artritis reumatoide. Toma hidroclorotiazida, atenolol,
dinitrato de isosorbide y aspirina. Al examinarle los ojos, se evidencia inyeccin
conjuntival y sequedad, la agudeza visual es 20/20 en ambos ojos. La funcin de los
msculos extraoculares y el examen de fondo de ojo, estn normales. Cul podra ser el
siguiente paso ms apropiado en el manejo de esta paciente?
A) Omitir hidroclorotiazida
B) Referencia inmediata al oftalmlogo
C) Hacer una prueba para cuantificar los anticuerpos anti Ro (SSA)
D) Prescribir lgrimas artificiales
E) Prescribir ungento de eritromicina
71) Una mujer de 65 aos, acude al mdico porque tiene una historia de prdida de peso
de 3 meses de evolucin, sensacin de quemadura en la lengua, fatiga, anorexia y un
dolor abdominal difuso. Aparenta estar plida y ella no puede sentir un diapasn que el
mdico le coloca en ambos dedos gordos del pie. Su hematocrito es de 35%. Se le realiza
una endoscopia alta que muestra unas rugosidades discretas a la inspeccin inicial. Se le
tomaron varias biopsias de estmago y de esfago y el informe de patologa, est
pendiente. Cul de las siguientes opciones es el hallazgo ms probable que se
encuentre en los estudios de laboratorio posteriores?
A) Prueba de estimulacin con secretina anormal
B) Sangre oculta en heces positiva
C) Displasia de alto grado en la biopsia
D) Niveles de gastrina muy elevados en plasma
E) Folato srico en 1.4 ng/ml (N: >1.9 ng/ml)
72) Una mujer de 67 aos, viene al cuatro de urgencias porque tiene secreciones nasales
constantes y acuosas y dolor de cabeza desde hace 5 das. Ella no ha tenido tos, fiebre,
escalofros o estornudos. Ha tomado loratadina en los ltimos 4 das, sin observar
mejora. Tiene historia de depresin, hipertensin y osteoporosis. Toma hidroclorotiazida,
suplementos de calcio y venlafaxina. Su temperatura es de 37C, Presin arterial de
138/76 mmHg, pulso de 88 x min y respiraciones de 18 x min. Al examen se le observa
una rinorrea acuosa y clara. Se le realiza una prueba de Beta-2 transferrina a la secrecin
nasal y da positivo. Cul de las siguientes opciones es la complicacin ms probable de
la condicin de la paciente?
A) Meningitis por Haemophyllus influenzae
B) Meningitis por Neisseria meningitidis
C) Meningitis por Staphylococcus aureus.
D) Meningitis por Staphylococcus epidermidis
E) Meningitis por Streptococcus pneumoniae
73) Una mujer de 63 aos, llega al consultorio quejndose de diarrea acuosa profusa. HA
tenido este sntoma desde hace varias semanas y a pesar de usar loperamida y cambiar
su dieta, la diarrea persiste, niega viajes recientes o excursiones para acampar. A pesar
26

que ella no coma, sigue con diarrea y adicional, se queja de rubor facial espordico. Su
historia mdica es relevante por hipertensin, para lo cual toma hidroclorotiazida desde
hace muchos aos. Al examen fsico, se ve deshidratada y tiene pobre turgencia en la
piel, se le ven las mucosas resecas, el resto del examen, sin complicaciones. Ella es
enviada para realizarle un estudio del intestino delgado con bario, que demuestra un
defecto de llenado no obstructivo en el yeyuno distal y en la tomografa, se ve un tumor
que protruye hacia la luz del intestino delgado. Dada la historia, cul de las siguientes
opciones, es ms probable que coexista con este tumor de intestino delgado?
A) Estenosis artica
B) Plipos hamartomatosos
C) Metstasis hepticas
D) Tumor medular de tiroides
E) Metstasis a ganglios linfticos mesentricos
74) Un hombre de 47 aos, con enfermedad lcero pptica, viene al mdico para su
consulta de seguimiento. l tuvo dolor epigstrico y se le diagnostic una lcera en el
duodeno e infeccin por H. pylori a travs de una endoscopia que se le realiz hace 2
meses. Ha disminuido el tamao de las comidas y dej de tomar caf y alcohol. Complet
su tratamiento de dos semanas con amoxicilina, claritromicina y omeprazol. No ha tenido
mejora de sus sntomas. Cul de las siguientes opciones, es el siguiente paso ms
apropiado en el manejo?
A) Bismuto, metronidazol y tetraciclina QID con omeprazol BID por 7 a 14 das
B) Evaluar niveles de gastrina en sangre
C) Realizar una prueba de aliento de urea
D) Omeprazol, levofloxacina y amoxicilina BID por 14 das
E) Repetir omeprazol, amoxicilina y claritromicina BID de 7 a 14 das
75) Un hombre de 19 aos, va a colonoscopia por una historia familiar de poliposis en sus
hermanos. A su hermano, a la edad de 23 aos y su hermana a los 29 aos, les
realizaron una proctocolectoma total. La colonoscopia muestra de 150 a 200 plipos
pequeos en el rectosigmoides. Se le realiza biopsia a 5 plipos y todos muestran
adenomas benignos. Cul de las siguientes opciones, es el siguiente paso ms
apropiado en el manejo?
A) Administrar 5-FU y leucovorina
B) Realizar biopsia de ganglio linftico
C) Reseccin del colon sigmoides
D) Programar para colonoscopia en un ao
E) Proctocolectoma total
76) Un hombre de 51 aos viene al cuarto de urgencias por fiebre, nuseas, vmitos y
dolor abdominal que se irradia al hombro derecho de dos das de evolucin, Su historia
mdica, no ha tenido complicaciones y no toma medicamentos. Su temperatura es de
38.3C, Presin artrial de 130/80 mmHg y pulso en 90 por min. Al examen fsico, se le
encuentra sensibilidad a la palpacin en el hipocondrio derecho y con una parada abrupta
de la inspiracin a la palpacin en ese mismo sitio. Tiene sus escleras anictricas. Cul
de las siguientes opciones, es el siguiente paso ms apropiado en el manejo?
A) Colecistectoma en las prximas 24 a 48 horas
B) Colecistectoma ahora
27

C) Antibiticos IV y observacin
D) Lquidos IV y observacin
E) Drenaje percutneo ahora
77) Una mujer de 18 aos, viene al mdico porque tiene irritabilidad extrema antes de su
regla. Durante ese periodo, ella se pone a llorar y no se puede concentrar. Su apetito
aumenta y se le antojan chocolates y dulces. Tambin tiene dolor de cabeza, mareos,
sensibilidad mamaria, distensin abdominal y sntomas gastrointestinales. Los sntomas
mejoran con la menstruacin y esto le ha pasado cada mes, por 4 meses. Cul de las
siguientes opciones, es el sntoma fsico ms comn en esta paciente?
A) Distensin abdominal
B) Sensibilidad mamaria
C) Labilidad emocional
D) Antojos de comida
E) Dolor de cabeza
78) Un hombre de 40 aos, viene al mdico por una historia de dolor punzante en sus
pies y ha tenido una decoloracin azul en los mismos. Tiene una intolerancia extrema al
fro y sus piernas se ponen plidas cuando las eleva. SU historia mdica no tiene
complicaciones y niega poliuria, polidipsia, fatiga, prdida de peso, dolor de pecho o
edema. HA fumado dos paquetes de cigarrillos diarios por 20 aos. Su Presin artrial es
de 129/74 mmHg y el pulso en 77 x min. La saturacin de oxgeno a aire ambiente es de
96%. Al examen fsico, la cabeza y el cuello no tienen anormalidades, no tiene
ingurgitacin yugular, linfadenopata o soplos carotdeos. Los pulmones estn claros a la
auscultacin. El corazn est normal y no se le escuchan soplos. No tiene sensibilidad a
la palpacin abdominal y tiene una lcera pequea en el pie derecho. Los laboratorios
muestran lo siguiente:
Na+: 141 mEq/L
HCO3-: 25 mEq/L
Glucosa: 87 mg/dl

K+: 4.1 mEq/L


BUN: 17 mg/dl
Hemoglobina A1C: 5.5%

Cl-: 105 mEq/L


Creatinina: 0.9 mg/dl

La angiografa de las extremidades inferiores, muestra un patrn tortuoso de las arteriolas


perifricas con afectacin segmentaria. Las arterias proximales estn conservadas. Cul
de las siguientes opciones, es el diagnstico ms probable?
A) Isquemia aguda de las extremidades
B) Anticoagulante lpico
C) Enfermedad arterial perifrica
D) Fenmeno de Raynaud
E) Tromboangeitis obliterans
79) Una mujer de 63 Aos, viene al cuarto de urgencias con dolor en el pecho, nuseas y
dificultad para respirar y disfagia aguda. El dolor se irradia a la parte izquierda de la
mandbula y hacia la espalda y no vara de intensidad ni se alivia con cambios de
posicin. Ella niega fiebre, escalofros y hemoptisis. Fue diagnosticada hace 7 aos de
hipertensin, la cual se trata con hidroclorotiazida. Sus signos vitales son, temperatura en
36.5C, Presin arterial en 160/92 mmHg, pulso en 115 x min y respiratorio en 26 x min.
Al examen fsico, se ve a una mujer anciana, muy angustiada por el dolor, se ve un
impulso apical con ritmo regular, sin frote y pulsos asimtricos en las extremidades
superiores. Los ruidos respiratorios estn disminuidos del lado izquierdo. El ECG muestra
28

una hipertrofia ventricular izquierda, sin alteraciones del segmento ST. Una radiografa de
trax muestra un ensanchamiento del mediastino de 9 cm y un botn artico nuboso.
Cul de las siguientes opciones es el mejor siguiente paso en el manejo de esta
paciente?
A) Labetalol
B) Nitroprusiato
C) Tomografa helicoidal con contraste
D) Tomografa helicoidal sin contraste
E) Ecocardiografa transtorcico
80) Una mujer de 62 aos con hipertensin y osteoporosis, viene al mdico para su cita
de control de cada 6 meses. Ella se queja de que se siente con nuseas y ha tenido un
leve malestar en el pecho en la ltima semana. Ella tambin se ha dado cuenta de que se
siente ms cansada de lo usual, manifiesta tambin que ha tenido dolores musculares.
Ella toma furosemida 40 mg BID y ocasionalmente, suplementos de calcio. Sus signos
vitales y el examen fsico, son normales. Un electrocardiograma, muestra la presencia de
una onda U y una depresin de <1 mm del segmento ST. La suplementacin con cul de
los siguientes, pudo prevenir esta condicin?
A) Calcio
B) Hierro
C) Magnesio
D) Manganeso
E) Potasio
81) Un hombre de 42 aos, se presenta en el consultorio por disnea de esfuerzo y tos
productiva. Se ha percatado de que estos sntomas han progresado en los ltimos meses.
Inicialmente el crea que haba adquirido una infeccin de las vas respiratorias
superiores, pero los sntomas han persistido. Niega historia de enfermedades anteriores y
no toma ningn medicamento, niega tabaquismo y se toma una copa de vino junto con las
comidas. Al examen fsico, tiene una temperatura de 38C, FC de 78 x min, FR de 16 x
min. Al examen de la cabeza y el cuello, no hay ingurgitacin yugular; el corazn tiene
ritmo regular y tiene un S2 intenso. Al examen pulmonar, tiene disminucin difusa de los
ruidos respiratorios; el abdomen est blando y no doloroso a la palpacin, los pulsos
perifricos estn normales. Se le ordena una espirometra y se le encuentra un patrn
obstructivo. Los laboratorios, muestran lo siguiente:
Na+: 137 mEq/l
K+: 4.1 mEq/l
Cl-: 105 mEq/l
HCO3-: 27 mEq/l
ALT: 115 U/L
AST: 130 U/L
Fosfatasa alcalina: 113 U/L
Cul de las siguientes opciones, es la explicacin ms probable para la elevacin de las
transaminasas en este paciente?
A) Acumulacin de alfa-1 antitripsina en los hepatocitos
B) Alcoholismo
C) Actividad aumentada de la alfa-1 antitripsina en el hgado
D) Hepatopata congestiva
E) Actividad desenfrenada de la elastasa heptica
82) Una mujer de 23 aos, G1P0, con un embarazo de 32 semanas, es trada al cuarto de
urgencias porque tuvo un accidente de trnsito y tiene un dolor abdominal intermitente.
29

Fue vctima de una colisin frontal porque un conductor borracho, invadi su carril. Su
Presin arterial es de 97/70 mmHg, pulso en 117 x min, respiratorio en 15 x min. Al
examen fsico est despierta y alerta y aparenta verse moderadamente adolorida. Al
examen plvico, se observa sangre en el perin. Tiene dolor a la palpacin abdominal. La
FCF est en 160 x min, con variabilidad disminuida. Cul de las siguientes opciones, es
el diagnstico ms probable?
A) Abrupto placentario leve
B) Abrupto placentario moderado
C) Placenta previa
D) Quiste de ovario roto
E) Abrupto placentario grave
83) Un hombre de 42 aos acude al mdico por evaluacin de rash cutneo y cada del
cabello. Tiene antecedente de Enfermedad de Crohn de larga evolucin y tuvo una
reseccin extensa del intestino delgado con el resultante sndrome de intestino corto.
Actualmente recibe nutricin parenteral total. Cuando trata de comer, se queja de que la
comida no sabe a comida. Sus signos vitales estn estables. Al examen muestra
alopecia y lesiones bulosas y pustulares alrededor de las reas perioral y periorbital.
Cul de los siguientes es la causa ms probable de los sntomas actuales del paciente?
A) Enfermedad celiaca
B) Deficiencia de zinc
C) Deficiencia de vitamina A
D) Deficiencia de vitamina B12
E) Deficiencia de selenio
84) Un hombre de 46 aos es hallado taquineico en su segundo da de reparacin de
hernia ventral en el abdomen superior. No tiene dolor torcico. La ciruga no tuvo
complicaciones, pero en la tarde el paciente se quej de dolor abdominal y solicit
calmantes. El tiene una tos crnica con algo de esputo matutino. Sus otros problemas
mdicos incluyen asma leve persistente, hipertensin e hiperlipidemia. El paciente es un
fumador activo de 25/paquetes /ao. Su temperatura es de 36.7C, presin arterial de
123/79mmHg, pulso de 90/min, respiracin de 28/min. Su ndice de masa corporal es de
32Kg/m2. El examen muestra disminucin de los ruidos pulmonares a la base derecha sin
sibilancias ni prolongacin de la espiracin. Los sonidos cardiacos estn normales. Hay
algo de distensin abdominal y leve dolor difuso sin rebote. La radiografa muestra una
opacidad densa en las bases pulmonares. La gasometra a aire ambiente reflejara cul
de las siguientes?
pH
A
B
C
D
E

7.30
7.30
7.41
7.49
7.49

PO2 (mmHg)
70
110
98
70
110

pCO2 (mmHg)
54
22
38
27
50

85) Un hombre de 59 aos es llevado a urgencias debido a confusin progresiva en los


ltimos 2 das. Ha tenido varios episodios de vmitos sin hematemesis. Segn la esposa,
se ha quedado de cefaleas severas y poco sueo recientemente. Seis meses antes, fue
30

diagnosticado con cncer prosttico localizado que fue tratado con ciruga. Sus
antecedentes mdicos incluyen: hipertensin, gota, cefalea tensional y asma leve
intermitente. No fuma pero su esposa ha notado que ha estado bebiendo 2 3 shots de
whiskey cada noche. No tiene alergias conocidas. Al examen fsico hay hepatomegalia.
No hay ascitis ni edema perifrico. Los laboratorios muestran:
Albmina: 3.2 mg/dl
Bilirrubina total: 6.7mg/dl
Aspartato aminotransferasa (AST, SGOT): 6245 UI/L
Alanino aminotransferasa (ALT, SGPT): 7116 UI/L
Tiempo de protrombina: 28 seg
Tiempo de tromboplastina: 43 seg
Cul de las siguientes es el diagnstico ms probable de este paciente?
A) Hepatitis alcohlica aguda
B) Cirrosis heptica
C) Dao heptico inducido por medicamentos
D) Malignidad metastsica a hgado
E) Esteatosis heptica no alcohlica
86) Un hombre de 32 aos es admitido al hospital por ser encontrado al borde de un
puente. Impresiona severamente deprimido, anunciando que su vida est acabada. El
est arruinado econmicamente y su esposa recientemente lo abandon, tom a sus hijos
y los llev con ella. El ha estado llorando, incapaz de dormir, sin apetito y continuamente
expresa sus sentimientos de desesperanza y pensamientos suicidas. Se le inici
sertralina y clonazepan para sus sntomas. Ms tarde en ese da, el paciente se vuelve
molesto e inicia golpendose la cabeza repetidamente contra el borde del lavamanos. Se
le administran inyecciones intramusculares de lorazepan y haloperidol para reducir su
agitacin y fue fsicamente restringido por un breve periodo de tiempo. El contina
agitado los das siguientes, requiriendo lorazepan y haloperidol de emergencia unas tres
veces ms y restriccin fsica unas 2 veces ms. En el tercer da, el paciente
notablemente ms calmo, es incapaz de salir de cama. Impresiona confundido y no
responde a las preguntas en forma apropiada. Est diafortica y su temperatura es
39.5C. Su presin arterial es 167/97mmHg y su pulso en 112/min. Como el personal trat
de sacarlo de la cama, notaron que sus brazos estn rgidos y difciles de doblar. Cul
de los siguientes es el ms probable causal de estos sntomas?
A) Episodios de golpearse la cabeza
B) periodos de restriccin fsica
C) Haloperidol
D) Sertralina
E) empeoramiento de los sntomas psiquitricos.
87) Un hombre de 45 aos con linfoma de Burkitt es tratado con una combinacin de
quimioterapia y alopurinol. Al 3er da de tratamiento, nota que tiene disminucin del gasto
urinario con aumento de los niveles de nitrgeno de urea y creatinina. Al paciente se le
inicia hidratacin intensa. El EKG muestra intervalos QT prolongados. Se sospecha de
sndrome de lisis tumoral. Se ordena un panel metablico completo. Cul de los
siguientes conjuntos de resultados se esperaran en este paciente?

31

Calcio

Fosfato

Potasio

cido rico

Alto

Alto

Alto

Alto

Bajo

Bajo

Bajo

Bajo

Bajo

Bajo

Alto

Alto

Bajo

Alto

Alto

Alto

Bajo

Alto

Bajo

Bajo

88) Un hombre de 69 aos se somete a ciruga de bypass arteria coronaria con reemplazo
de vlvula artica. El procedimiento fue exitoso y el paciente es extubado y pasado de
unidad cuidados intensivos a sala intermedia en el da postoperatorio 2. Esa noche, se
queja de debilidad, opresin torcica y dificultad respiratoria. Su presin arterial es 70/30
mmHg, respiracin de 26/min y la frecuencia cardiaca es de 148/min. La auscultacin
pulmonar revela crpitos bibasales. El EKG se muestra a continuacin:

Cul de las siguientes es el mejor prximo paso en el manejo de este paciente


A) Amiodarona
B) Marcapaso transcutneo
C) Cardioversin elctrica
D) Lidocana
E) Digoxina
89) Una mujer de 55 aos acude al mdico por quejarse de 2 aos de tos episdica y
produccin de esputo amarillo. Su primer episodio dur alrededor de 3 semanas con
congestin del pecho, tos productiva y esputo purulento con disnea. Ella vio a su mdico
de atencin primaria en ese momento y la diagnostic con bronquitis y le recet
antibiticos. La paciente desde entonces ha tenido 6 episodios adicionales, cada uno con
gran cantidad de esputo amarillo (algunas veces teidos de sangre), disnea y congestin
a nivel de senos paranasales. Ella mejora en cada ocasin con antibiticos. Sin embargo,
ella contina teniendo tos diaria y se ha empeorado en las ltimas 2 semanas. No tiene
fiebre, escalofros, dolor torcico, mascotas en casa ni ha viajado recientemente. Tiene
antecedente de tabaquismo de 30 paquetes/ao. Su temperatura es 37.2C, presin
arterial de 120/68mmHg, pulso en 80/min y respiratoria en14/min. Su cuello es libre de
adenopatas. En los pulmones se escuchan roncus difusos con sibilancias y algunos
crpitos gruesos en las bases. El resto de la evaluacin no muestra anormalidades. La
32

radiografa muestra lineales atelectasias en los campos pulmonares medios e inferior


bilateralmente.

Cul de los siguientes es el prximo paso ms apropiado para confirmar el diagnstico


de esta paciente?
A) Broncoscopia con lavado alveolar
B) Ecocardiografa
C) tomografa torcica de alta resolucin
D) Prueba de funciones pulmonares
E) Esputo por Bacilo alcohol cido resistente
90) Un estudio es realizado para evaluar una nueva prueba serolgica para HIV. 1000
personas fueron seleccionadas al razn de la poblacin, se evaluaron utilizando la nueva
prueba y nuevamente utilizando la prueba estndar (por ejemplo el Western Blot) para
determinar su real estatus infeccioso. Los hallazgos fueron los siguientes:

33

Cul es la sensibilidad de esta nueva prueba serolgica?


A) 140/180
B) 140/200
C) 60/820
D) 60/200
E) 180/100
91) Un hombre de 23 aos de edad, previamente sano se presenta al servicio de
urgencias tras sufrir una herida de bala en el pecho izquierdo. La herida de entrada est 3
cm debajo del pezn y la herida de salida es justo debajo de la escpula. Un tubo torcico
se coloca y drena 400 ml de sangre y contina drenando 50-75 ml / h durante la
reanimacin inicial. La presin arterial inicial de 70/0 mmHg responde a 2 litros de
cristaloides y ahora est 100/70 mmHg. El examen abdominal sin nada especial. La
radiografa de trax revela un pulmn reexpandido y no hay aire libre bajo el diafragma. El
siguiente paso en el manejo debe ser:
A) Admisin y observacin
B) Lavado peritoneal
C) Toracotoma exploratoria
D) Laparotoma exploratoria
E) Exploracin de la herida local
92) Una mujer de 59 aos, G4P4, se presenta a su oficina por de perder orina cuando
tose, estornuda, o que realicen determinados tipos extenuantes de actividad fsica. El
problema se ha vuelto cada vez peor en el ltimo ao, hasta el punto donde el paciente
encuentra sus actividades de la vida diaria comprometidas por miedo a la vergenza.
Ella niega cualquier otro sntoma urinario como urgencia, frecuencia, o hematuria. Niega
cualquier problema con sus movimientos intestinales. Sus cirugas previas incluyen una
amigdalectoma y apendicectoma. Tiene Antecedentes de Diabetes tipo 2, con glicemias
bien controlados con antidiabticos orales. No tiene antecedentes de problemas
ginecolgicos en el pasado. Tuvo 4 hijos por parto vaginales espontneos; sus pesos
oscilado entre 8 y 9 libras. Actualmente es sexualmente activa con su pareja de 25 aos.
Hace 4 aos con menopausia y nunca ha tomado terapia de reemplazo hormonal. Su
altura es de 167cm., peso de 190lbs Su presin arterial es de 130/80 mmHg. Basados
en los antecedentes de la paciente, cul es el diagnstico ms probable?
A) Incontinencia por rebosamiento
B) Incontinencia de esfuerzo
C) Infeccin del tracto urinario
D) Inestabilidad del detrusor
E) Fstula vesicovaginal
93) Un hombre de 70 aos acude a urgencias quejndose de grandes hematomas y
sangrado en las encas. Entre sus antecedentes mdicos est la fibrilacin atrial tratada
con metoprolol y warfarina. l ha estado estable en su dosis actual de warfarina por
varios meses. Adicional toma acetaminofn para el dolor articular cuando se ejercita
mucho. Ha estado tomando la warfarina como fue prescrito y ha tratado de mantener un
estilo de vida saludable con ejercicio y comiendo ms vegetales. Su nueva dieta consiste
principalmente en vegetales como espinacas, col de Bruselas, brocoli y grandes
cantidades de t verde. Cul de los siguientes cambios es el responsable por el
sangrado de este paciente?
34

A) Acetaminofn
B) Col de Bruselas
C) Ginseng
D) Aumento de ejercicio
E) Espinaca
94) Un hombre de 60 aos es llevado a urgencias por su hijo debido a un episodio de
sncope. l estaba viendo la televisin en su silln cundo de repente perdi la conciencia
sin ningn signo de alarma. Tuvo varias sacudidas clnicas mientras estaba inconsciente.
Nunca tuvo un episodio previo similar. Sus antecedentes mdicos son importantes por
infarto agudo al miocardio hace 6 meses e hipertensin bien controlada. Sus actuales
medicamentos incluyen captopril, metoprolol, hidroclorotiazida, clopidogrel y aspirina. Su
presin arterial es de 130/85 mmHg y su frecuencia cardiaca es de 80/min con frecuentes
latidos ectpicos. Al examen fsico se revela un solo leve holosistlico apical irradiado a
axila. Cul de los siguientes es el mecanismo patofisiolgico ms probable del sncope
del paciente?
A) Reaccin vaso vagal
B) Arritmia
C) Disfuncin autonmica
D) Convulsin
E) Hipotensin postural
95) Un hombre de 55 aos acude a urgencias por debilidad del lado izquierdo que ha
persistido por varias horas. Tambin se queja de un fuerte dolor en el centro del trax tipo
desgarrador que se irradia a espalda. El es hipertenso desde hace 7 aos y no ha estado
bajo tratamiento antihipertensivo. Fuma un paquete de cigarrillos das por los ltimos 25
aos. Su padre falleci por enfermedad coronaria a la edad de 44 aos. Su temperatura
es de 37.1C, pulso de 78/min y presin arterial de 180/120mmHg en el brazo derecho y
110/70mmHg en el brazo izquierdo, respiracin de 16/min. Est diafortico y confundido.
Su evaluacin neurolgica es significativo por disminucin de la fuerza y tono en las
extremidades superiores e inferiores. El resto del examen fsico es normal. EKG muestra
desviacin del eje a la izquierda y la radiografa de trax es normal. Basado en estos
hallazgos, cul es el diagnstico ms probable?
A) Hemorragia subaracnoidea
B) Evento cerebrovascular hemorrgico
C) Embolismo pulmonar paradjico
D) Infarto miocrdico con Tromboembolismo
E) Diseccin artica
96) Una mujer de 65 aos es admitida al hospital por aumento progresivo de la disnea,
aumento de peso y edema de las extremidades inferior. Tiene antecedente de
hipertensin, cardiomiopata no isqumica con fraccin de eyeccin del 30% e
hiperlipidemia.
Sus
medicamentos incluyen aspirina oral, digoxina, furosemida,
metoprolol, lisinopril y atorvastatina. Se le inicia furosemida Intravenosa. En el da 3 de
hospitalizacin el monitoreo revela 6 latidos de taquicardia ventricular complejos anchos.
Al examen fsico muestra disminucin del edema de miembros y pulmones claros. Cul
de los siguientes es el paso prximo ms apropiado en el manejo de la taquicardia de
esta paciente?

35

A) Aadir espironolactona
B) Aadir metolazona
C) Medir electrolitos sricos
D) Descontinuar atorvastatina
E) Descontinuar metoprolol
97) Un hombre de 59 aos sufre un infarto al miocardio. Es tratado mdicamente y dado
de alta a los 10 das con aspirina, atorvastatina, metoprolol, lisinopril y nitroglicerina
sublingual. Un mes ms tarde, el acude a visita de control, niega dolor torcico, disnea o
vrtigo. Su presin arterial es 120/75 mmHg. Su EKG es el mostrado a continuacin:

El Ecocardiograma muestra una fraccin de eyeccin del 45%. Cul el es el mejor


prximo paso en el manejo de este paciente?
A) Observacin
B) Amiodarona
C) Digoxina
D) Cardioversin sincronizada
E) Verapamilo
98) Un hombre de 45 aos acude a evaluacin por diarrea crnica. El ha perdido casi 7Kg
(15lbs) en el ao pasado. No hay sangre en las heces. La medicin de heces de 24 horas
revela contenido de grasa fecal de 10g/d (normal es <6g/d). La microscopia no muestra
patgenos ni leucocitos. Electrolitos y funcin renal estn en lmites normales. El
paciente se le da 25g de solucin oral de D-Xylosa y su excrecin urinaria a las 5horas de
D-Xylosa es de 1.2g (lo normal es de 4.5-7.5g). Se repite la prueba en 4 semanas de
tratamiento con antibiticos y la excrecin urinaria de D-Xylosa es de 1.3g. Basado en
estos hallazgos, cul de los siguientes es el diagnstico ms probable?
A) Sobrecrecimiento bacteriano
B) Enfermedad celiaca
C) Intolerancia a la lactosa
D) Insuficiencia pancretica
E) Enfermedad ileo terminal
99) Un hombre de 29 aos con historia de infeccin por HIV de 6 aos se presenta con
diarrea severa crnica asociada a malestar general, nuseas, anorexia y calambres
abdominales. Su ultimo CD4 fue de 80 cel/mm3. Una tincin modificada alcohol cido
muestra en heces quistes de 4-6mm. Cul de los siguientes es el microorganismo ms
probable responsable de la condicin del paciente?
A) Micobacterium avium complex
B) Cryptosporidium parvum
C) Isospora belli
D) Pneumocystis jirovecii
E) Microsporidia
36

100) Un hombre de 63 aos acude por fatiga fcil. Tiene antecedente de hipertensin
tratada con lisinopril. En los ltimos 3 meses, ha tenido dolor de rodilla derecha debido a
osteoartritis y ha estado tomando naproxeno para el alivio del dolor. Su presin arterial es
123/78 mmHg, frecuencia cardiaca 78/min. Su rodilla derecha est levemente deformada
debido al sobrecrecimiento seo pero no hay dolor o edema. Al examen fsico no hay
otros hallazgos relevantes. Los resultados de laboratorio son:
Hemoglobina 8.6mg/dl
VCM: 72 fl
Leucocitos: 7,200/mm3
plaquetas: 165,000/mm3.
Creatinina 1.2 mg/dl
VES: 15 mm/hr
Cul de los siguientes cambios son los que usted esperara tuviera el paciente?
Hierro srico

Ferritina

Capacidad fijacin del


hierro

Saturacin
transferrina

Bajo

Alto

Bajo

bajo

Bajo

Bajo

Alto

bajo

Bajo

Bajo

Bajo

alto

Alto

Alto

Bajo

alto

Normal

Normal

Normal

normal

101) Una mujer de 25 aos acude por disnea de ejercicio y fatiga. No tiene antecedentes
patolgicos relativos. No fuma ni consume alcohol. Su presin arterial es 110/70 mmHg y
frecuencia cardiaca es de 90/min. Al examen fsico hay una conjuntiva plida. Los
laboratorios obtenidos son: VES: 15mm/hr; Hemoglobina 7.5g/dl; VCM: 70 fl; MCHC: 29%;
Glbulos blancos: 7,000/mm3, Neutrfilos segmentado: 55%; Eosinfilos: 3%, Bandas 3%
Basfilos 0%, Lifnocitos: 32%, Monocitos 7% Cul de los siguientes es el siguiente mejor
prximo paso en el manejo de la paciente?
A) Muestra de mdula sea
B) Estudios de hierro
C) Niveles de folato srico
D) Prueba de Schilling
E) Medicin de HbA2
102) Un joven de 25 aos acude a consulta con dermatologa por un rash. Este inicio
como lesin nica en el pecho pero a crecido, otras lesiones se han desarrollado tia. Al
examen fsico se observan mltiples lesiones descamativas en su pecho y espalda. La
mayor de esta, la cual refiere el paciente es la primera, mide 5cm en el dimetro, ovalada,
color rosado, con bordes ligeramente elevados, muchas placas pequeas, cerca de 1cm
de dimetro con similar apariencia se observan. Cul de los siguientes es el diagnstico
ms probable?
A) Molusco contagioso
B) Pitiriasis rosa
C) Pitiriasis rubra pilaria
D) Roscea
E) Escabiasis
37

103) Un hombre de 25 aos acude al mdico por instauracin rpida de dolor y edema de
su rodilla izquierda, que inici hace apenas 24 horas. Su temperatura es de 38.5C,
presin arterial de 125/70mmHg, pulso de 95/min y respiracin de 20/min. En su historia
personal refiere uso de drogas IV. Niega historia de enfermedades de transmisin sexual.
La rodilla izquierda est dolorosa, edematizada y caliente al tacto. La auscultacin
torcica es normal. Cul de las siguientes es el prximo paso en el manejo?
A) Laboratorios en sangre, incluyendo hemograma completo
B) Prueba de HIV antes de iniciar cualquier tratamiento
C) Rayos X simple de rodilla
D) Antiinflamatorios no esteroidales y tratamiento antibitico emprico
E) Artrocentesis diagnstica
104) Un veterano de 45 aos de edad, es visto en la clnica por su pobre control de los
impulsos. En el ltimo mes, ha invertido todos sus ahorros en empresas de negocios y
actualmente est distanciado de su esposa. l admite una menor necesidad de sueo, un
nuevo sentido de grandiosidad, y aumento de la energa dedicada a la investigacin de
proyectos para hacer dinero. l no tiene antecedentes psiquitricos previos, y su nica
historia clnica incluye el tratamiento de gonorrea durante su servicio militar y una
apendicectoma a la edad de 36 aos. El examen fsico revela un hombre delgado, de
mediana edad con el habla apresurada. Su examen pupilar es normal, excepto que las
pupilas son reactivos a la convergencia, pero no a la luz. La infeccin con cul de los
siguientes organismos podra ser la causa ms probable de sus sntomas?
A) Chlamydia trachomatis
B) Virus Herpes simple
C) Neisseria gonorrhoeae
D) Toxoplasma
E) Treponema pallidum
105) El nio de la imagen naci pequeo para la edad gestacional, con defencto
ventricular septal a la evaluacin cardiolgica. La apariencia del nio y sus hallazgos son
consistentes con:

38

A) Exposicin perinatal a fenitona


B) Trisomia 21
C) Sndrome de Alport
D) Sndrome alcholico fetal
E) Nio de madre diabtica
106) Una mujer blanca de 28 aos acude a la consulta por un examen de piel de rutina. Al
examen usted encuentra una lesin de color oscuro. La paciente refiere que ella tena un
lunar all toda su vida. Ella refiere que ha estado recientemente sintiendo comezn y
dolor algunas veces en el lugar del lunar. Ella refiere historia de quemaduras de sol
durante la niez y refiere que ella es muy sensible a la luz solar. La imagen del lunar es
la mostrada a continuacin:

Cul de las siguientes es el diagnstico ms probable?


A) Keratoacantoma
B) Nevo Azul
C) Melanoma
D) Nevo Melanoctico
E) Lntigo Simple
107) Un hombre de 21 aos acude a urgencias luego de recibir 3 heridas de arma de
fuego en su pierna. Cuando llega a urgencias se encuentra letrgico, dbil y confuso. Su
temperatura (37C), presin arterial de 80/40 mmHg, pulso de 143/min y respiracin de
28/min. Se le administran varios litros del fluido IV y 4 unidas de glbulos rojos
empacados y su presin arterial aumenta 100/74 mmHg. Es llevado a la unidad de
cuidados crticos para observacin. Los estudios de laboratorio se muestran:
Sodio: 145 mEq/L
Bicarbonato: 26mEq/L
Creatinina: 1.7mg/dl
Plaquetas: 270,000/mm3

Potasio: 4.5 mEq/L


Glucosa: 116 mg/dl
Hemoglobina: 7g/dl
Hematocrito: 21%

Cloruro: 90 mEq/L
Nitrogno Urea: 77mg/dl
Leucos: 5,700/mm3

24 horas ms tarde su temperatura es 37C (98.6F), presin arterial de 110/70mmHg,


pulso de 83/min y respiracin de 18/min. Los laboratorios ahora son:
Sodio: 148 mEq/L
Potasio: 4.9mEq/L
Cloruro: 100 mEq/L
Bicarbonato: 23 mEq/L
Glucosa: 146 mg/dl
Nitrgeno de Urea: 57mg/dl
Creatinina: 2.6 mg/dl
Hemoglobina: 11g/dl
Leucos: 5,700/mm3
Plaquetas: 270,000/mm3
Hematcrito: 29%
CPK: 1400 U/L
39

Urinlisis: Color: marrn oscuro


Osmolalidad: 155 mOsm/Kg
Nitrito: Negativo
Sangre: Negativo
Eosinfilos: Ninguno

Gravedad especfico: 1020


Estearasa leucocitaria: Negativo
Protena: trazas
Microscopio: cilindros granulares oscuros
Sodio urinario: 50 mEq/L

Cul de los siguientes es la etiologa ms probable de la falla renal de este paciente?


A) Necrosis Tubular Aguda
B) Nefritis intersticial Alrgica
C) Azohemia Pre-renal
D) Glomerulonefritis rpidamente progresivamente
E) Rabdomilisis.
108) Un hombre de 62 aos un hombre regresa a cita de seguimiento con su mdico. El
ha sido recientemente tratado con anfotericina B por neumona por Aspergillus, sin
embargo, an se queja de dificultad respiratoria y debilidad generalizada. Tiene una larga
historia de Enfermedad pulmonar obstructiva crnica (EPOC) tratado con albuterol /
Ipratropio en inhalador dosis medida, hipertensin tratado con hidroclorotiazida y lisnopril
y Enfermedad por reflujo gastroesofgico con omeprazol. Sus signos vitales son:
Temperatura: 36.9C (98.4F), pulso en 104/minuto, presin arterial 135/75 mmHg,
respiracin son 24/min, saturacin oxgeno: 94% en aire ambiente. El examen fsico
revela un adulto mayor sin distres agudo, taquicardia leve, Auscultacin pulmonar muestra
crpitos en el lbulo superior derecho ocasionalmente sibilancias bilaterales. Radiografa
de Trax muestra mejora y resolucin del infiltrado en lbulo superior derecho. Los
estudios de laboratorio se muestran:
pH: 7.28
PaCO2: 44mmHg
PaO2: 84mmHg
HCO3-: 14 mEq/L
Sodio: 137 mEq/L
Potasio: 3.1 mEq/L
Cloruro: 112 mEq/L
Creatinina: 1.5 mg/dl
Nitrgeno Urea: 28 mg/dl
Glucosa: 92 mg/dl
Calcio: 9.2 mg/dl
Urinlisis muestra pH 7.0 y gravedad especfico de 1020 y negativo por glucosa, cetonas,
glbulos rojos y glbulos blancos. Cul de las siguientes es la causa ms probable del
acidosis en este paciente?
A) Acidosis Tubular Renal tipo 1
B) Acidosis Tubular Renal Tipo 2
C) Acidosis Tubular Renal Tipo 4
D) Insuficiencia respiratoria
E) Efecto adverso de Lisinopril
109) Un hombre de 45 aos acude a urgencias con letargia. En las ltimas 2 semanas ha
estado con episodios de abundante diarrea acuosa mltiples veces al da asociado a
vmitos no biliosos. 2 semanas atrs el pas tiempo con sus sobrinas y sobrinos, quienes
se enter poco despus que todos ellos tenan algn tipo de Bicho. El ha tenido
dificultad reteniendo todo, y en consejo de su mdico de atencin primario finalmente
decidi acudir a urgencias para mayor evaluacin. En emergencias sus signos vitales
son: presin arterial de pie: 90/60 mmHg, y pulso de 80/min; mientras que acostado la
presin arterial es 70/50 mmHg y pulso es 120/min. El paciente presenta debilidad y
malnutricin. Su piel tiene la turgencia disminuida, membranas mucosas secas a la
40

observacin, y sus ojos impresionan hundidos. Estudios de laboratorio se ordenan y se


observan lo siguiente:
Glbulos blancos: 8,000/mm3
Hemoglobina: 15g/dl
Hematocrito: 45%
Plaquetas: 250,000/mm3
Sodio: 150 mEq/L
Potasio: 3.3 mEq/L
Cloruro: 98 mEq/L
Bicarbonato: 30 mEq/L
Nitrgeno de urea: 30 mg/dl
Creatinina: 1.0 mg/dl
Glucosa: 70 mg/dl. Cul es el paso ms apropiado en el manejo de la condicin actual
de su paciente?
A) Reemplazo de electrolitos por suero oral por va oral
B) Agua Libre intravenosa
C) Solucin Salina 0.45% intravenosa
D) Solucin Salina 3% intravenosa
E) Solucin Salina 0.9% Intravenosa
110) Un hombre es llevado a urgencias por falta de respuesta a estmulo. Es conocido
alcohlico y fue encontrado en el suelo de su apartamento. A la evaluacin, est
deshidratado y sin respuesta. Las pruebas de laboratorio revelan lo siguiente:
Glbulos rojos: 4.5mill/mm3
Glbulos blancos: 4,500/mm3
Calcio: 10.8mg/dl
Creatinina: 3.5mg/dl

Hemoglobina: 12 mg/dl
Potasio: 5.8 mg/dl
Nitrgeno urea: 50mg/dl
Creatinin Kinasa: 15,000 U/L

Urinlisis en tira reactiva:


Gravedad especfica: 1020
Sangre: 3+
Urinlisis microscpico:
Glbulos rojos: 0-2 cpc
Glbulos blancos: 0-5 cpc
Cilindros: cilindros pigmentados presentes
Cul de los siguientes es el diagnstico ms probable?

Hematocrito: 36%
Fosfato: 2.2mg/dl
LDH: 489 U/L

Protena: 1+

A) Glomerulonefritis aguda
B) Nefritis intersticial alrgica
C) Hemlisis intravascular
D) Falla renal aguda pre-renal
E) Rabdomilisis.
111) Un paciente de 45 aos en tratamiento con hemodilisis por falla renal crnica ha
notado cada vez ms dificultad para control de su presin arterial. Refiere que tiene buena
adherencia con sus medicamentos que incluyen: eritropoyetina, sulfato ferroso,
vancomicina y vitamina D. Su presin arterial es de 180/99 mmHg. Cul de las
siguientes es la causa ms probable del empeoramiento en el control de su presin
arterial?
A) Eritropoyetina
B) Sulfato Ferroso
C) Vancomicina
D) Vitamina D
E) Uremia
41

112) Un infante masculino de 2 meses es llevado por sus padres luego de un intento de
alimentarlo tuvo un episodio de vmito n bilioso en proyectil. Sin embargo tiene sonidos
intestinales normales. Los padres son de un pas tercer mundo donde tuvo mnimo
control prenatal, y como resultado el nio naci prematuro por 6 semanas. A la
evaluacin fsica, el nio tiene fontanelas deprimidas, mucosa oral seca, y aumento del
pliegue cutneo y a la palpacin del abdomen hay una masa en forma de oliva reducible
en el cuadrante superior derecho. Los resultados de laboratorio son:
Sodio: 145 mEq/L
Potasio: 3.3. mEq/L
Cloruro: 94mEq/L
Bicarbonato: 38mEq/L
Creatinina: 1.0mg/dl
Nitrgeno Urea: 20mg/dl
Glucosa: 60mg/dl
pH: 7.50
PaCO2: 50mmHg
PaO2: 90mmHg. Cul es el manejo ms apropiado para este paciente?
A) Ultrasonido abdominal
B) Endoscopia
C) Reemplazo de cido hidroclrico
D) Solucin intravenosa de Sodio 0.9%
E) Laparotoma con piloroplasta
113) Un nio de 5 aos es llevado al pediatra 8 horas despus de caer de su bicicleta y
golpear la cabeza contra la acera. No hubo testigos en e accidente. El paciente niega
prdida de la conciencia. El nio es por lo dems sanos. Est al da con sus
inmunizaciones y no toma medicamentos. A la evaluacin fsica sus signos vitales son
estables. Hay una abrasin de 5 x 4 cm en su frente. El est alerta, orientado en
persona, lugar y tiempo. Su evaluacin sensorial y motora es normal y reflejos normales.
Cul de las siguientes constituye le manejo ms razonable para la situacin?
A) Admitir para observacin durante la noche
B) Instruir a familiares que observen el estado neurolgico por 24 horas
C) obtener una tomografa computada de cabeza
D) Obtener radiografa de crneo y dar de alta si es normal
E) obtener radiografa de crneo y observar por 24 horas luego dar alta.
114) Un hombre de 60 aos es admitido al hospital debido a dificultad respiratoria, mareo
y dolor torcico irradiado a su brazo izquierdo por 90 minutos. Tiene antecedentes de
diabetes mellitus tipo 2 e hipertensin arterial. Sus actuales medicamentos incluyen:
hidroclorotiazida y metformina. Bebe de 5 a 8 cervezas diarias y ha fumado un paquete
de cigarrillos diario por 38 aos. Su presin arterial es 85/60mmHg y pulso en 35/min. A
la evaluacin no hay distensin venosa yugular. Los pulmones estn claros a la
auscultacin. Los laboratorios muestran elevacin de la troponina I y de la Creatinin
kinasa. El EKG muestra elevacin del ST en derivadas II, III y avF. EKG derecho
muestra elevacin del ST en V4 y V5. Es admitido y se le administra nitroglicerina,
metoprolol y heparina. Cul de los siguientes es el primer paso ms apropiado para el
manejo del paciente?
A) Administrar dobutamina
B) Administrar lquidos IV
C) Cateterismo cardiaco con angioplastia
D) Insertar un cable de marcapaso temporal
E) Detener el metoprolol

42

115) Una nia de 4 aos es encontrada bebiendo una botella de limpiador de drenaje
lquido e inmediatamente es llevada a urgencias. Ella se muestra muy irritable y no desea
tragar ningn medicamento lquido.
Su madre refiere que ha estado llorando
intensamente, que ha estado con nuseas pero no ha vomitado. Evaluacin de la cavidad
oral no muestra evidencias de quemaduras o ulceraciones. Cul de los siguientes es el
manejo ms apropiado para el caso?
A) Trago de Bario
B) Tomografa computada de abdomen
C) Nada por boca por 12 horas, luego dieta de lquidos claros por 3 das
D) Larignoscopia indirecta
E) Esofagoscopa
116) Un hombre de 37 aos, de 70 Kg (154libras) es llevado a urgencias debido a
quemaduras extensas de tercer grado. Luego de realizar los clculos, se le inicia solucin
de lactato de ringer. En las ltimas 3 horas su gasto urinario ha reportado 15ml, 22ml y
18 ml. Se verifica la posicin de la sonda Foley y su libre paso de drenaje orina. La
evaluacin fsica revela extensas quemaduras de tercer grado sobre el pecho, espalda y
muslos. Su temperatura es 37.7C (100F) presin arterial de 100/70mmHg, pulso en 98 /
min, saturando 98% y presin venosa central de 2cm H2=. Los estudios de orina inicial
muestran:
Apariencia: amarillo oscuro
Gravedad especfica: 1045
pH: 6.5
Glbulos blancos: 2-3 cpc
Glbulos rojos: Negativos
Cilindros: Negativo
Nitritos: Negativo
Sodio Urinario: 10mEq/L
La tira de orina es negativa por glucosa, bilirrubina y hemoglobina. La creatinina srica
est en 1.3mg/dl y el nitrgeno de urea en 30mg/dl. Cul de los siguientes es el prximo
paso ms apropiado en el manejo?
A) Administrar furosemida 40mg IV STAT
B) Administrar solucin salina ms bicarbonato de sodio
C) Aumento en la velocidad de administracin de lquido
D) Observar y repetir laboratorios en 3 horas
E) Realizar hemodilisis
117) Un hombre de 40 aos es llevado a urgencias por sus amigos. Refieren que tom
muchas pastillas tratando de suicidarse. Ellos no tienen consigo ningn recipiente de
medicamentos. Su temperatura es 39.3C ( 103F) presin arterial 90/55mmHg, pulso
110/min e irregular, respiracin de 22/min. Evaluacin fsica del paciente es orientado en
persona y lugar pero no en tiempo. Su piel est enrojecida y seca. La evaluacin
neurolgica muestra pupilas dilatadas. Electrocardiograma muestra Complejos QRS
ensanchados. Los estudios de laboratorio muestran AST 26 IU/L y ALT 18 IU/L. La
gasometra arterial muestra pH: 7.41. Cul de las siguientes sustancias es la ms
probable causa de intoxicacin?
A) Acetaminofn
B) Alprazolam
C) Clonidina
D) Desipramina

43

118) Un nio de 8 aos es llevado a urgencias por disminucin del estado mental. Su
madre refiere que ha notado que ha estado bebiendo lquidos y orinando con mayor
frecuencia en las ltimas semanas. Hoy se ha dificultado levantarlo en la maana y se ha
quejado de dolor abdominal. A la evaluacin fsica hay un varn somnoliento, afebril, con
vas areas despejadas y leve taquicardia. Membranas mucosas secas y labios
resquebrajados. El abdomen es moderadamente doloroso a la palpacin difusa pero no
hay defensa ni rebote. Los estudios de laboratorio muestran glicemia en 560mg/dl y
potasio en 5.9 mEq/L. La gasometra arterial muestra un pH en 7.18. El urinlisis revela
cetonas y glucosa positiva. La tomografa de abdomen es normal. La radiografa de trax
est sin patologa aguda. 2 horas despus de iniciar el tratamiento el mdico aade
potasio a los lquidos intravenosos del paciente. Cul de los siguientes explica mejor la
decisin teraputica del mdico?
A) Acidosis produce deplecin extracelular de potasio
B) Hiperglicemia produce intercambio de potasio al espacio extracelular
C) Hiperkalemia protege al paciente contra disrritmias
D) Hipokalemia es el resultado de la correccin de la acidosis
E) Potasio no debe ser aadido a los lquidos intravenosos.
119) Una mujer de 32 aos es llevada a emergencias debido a que su esposa la encontr
con mnima respuesta al llegar a casa del trabajo. Su temperatura es 37C (98.6F),
presin arterial 105/65mmHg, pulso 65/min y respiracin 14/min. Ella est sedada y
capaz de pronunciar en pocas frases, habla es arrastrando palabras como embriagada.
Sus antecedentes mdicos son significativos por asma desde la infancia y trastorno de
pnico hace 4 aos en tratamiento. Cul de los siguientes es el prximo paso ms
apropiado?
A) Flumazenil inmediatamente
B) Haloperidol inmediatamente
C) Lorazepam
D) Monitorear respiracin y conciencia
E) Naloxona
120) Un hombre de 27 aos es llevado a urgencias debido a heridas penetrantes en trax
y abdomen por ser apualado varias veces con un pica hielo largo. Al momento de la
admisin presenta un neumotrax derecho, para lo cual se le realiza colocacin de tubo
torcico antes de someterlo a anestesia general para laparotoma exploratoria. La ciruga
revela que no hay lesiones intraabdominales y termina antes de lo esperado. El paciente
se mantiene intubado, mientras se espera que el anestsico se disipe. Debido a que no
est moviendo suficiente aire, el paciente es colocado en ventilacin mecnica. De
pronto, el paciente entra en paro cardiaco y fallece. Durante todo el tiempo el paciente
estuvo hemodinmicamente estable y nunca present signos de hipotensin o arritmias.
Cul de los siguientes es la causa ms probable del paro cardiaco de este paciente?
A) Embolismo areo
B) Embolismo graso
C) Infarto miocrdico
D) Embolismo pulmonar
E) Neumotrax a tensin

44

121) Un hombre de 50 aos acude al mdico debido a historia de 6 meses de


convulsiones. No tiene aura o instauracin focal de la actividad convulsiva. Ha tenido un
total de 3 convulsiones, con movimientos tnicos clnicos generalizados que duran hasta
1-2 minutos acompaados de incontinencia urinaria. Tiene periodos de confusin de
hasta 15 minutos luego de cada convulsin. No toma ningn medicamento. Pero empez
a beber 15 a 18 cervezas dos veces a la semana desde su divorcio hace 7 meses atrs.
A la evaluacin neurolgica no muestra anormalidades. Cul de las siguientes sera la
explicara mejor la etiologa de las convulsiones de ese paciente?
A) Alergia a medicamentos
B) Antecedente neonatal del paciente
C) Profesin del paciente
D) Convulsiones relacionadas al uso de alcohol
E) La hora del da en la que la convulsin ocurre
122) Un hombre de 18 aos es llevado en estado letrgico a urgencias por sus padres
luego de que lo encontraron en el garaje de la casa hace 30 minutos. Cuando sus padres
lo encontraron haban muchas latas / envases abiertas sin marca junto a l. Es incapaz
de obtener mayor informacin por el paciente. Su temperatura es 36.8C (98.2F),
presin arterial de 90/50 mmHg, pulso de 100/min, respiracin de 30/min. A la evaluacin
fsica hay dolor abdominal difuso a la palpacin. Los resultados de laboratorio muestran:
Sodio: 142 mEq/L
Potasio: 3.0mEq/L
Cloro: 94 mEq/L
Bicarbonato: 6mEq/L
Nitrgeno de urea: 12 mg/dl Creatinina: 1.5mg/dl
Glucosa: 60mg/dl
pH: 7.25
PaCO2: 25mmHg
PaO2: 85mmHg
El urinlisis muestra: Color amarillo oscuro
pH: 4.5
Glbulos blancos: 2-5 cpc
Glbulos rojos: 3-4 cpc
Cetonas: Negativo
Glucosa: Negativo Nitritos: Negativo
Cristales: cristales monohidrato forma de aguja.
Su estado mental no mejora al tratarlo con glucosa y /o naloxona. Cul de los siguientes
es el prximo paso ms apropiado en el manejo de este paciente?
A) Dosis de carga de etanol de 800 mg/Kg
B) Dosis de carga de Fomepizole 15 mg/Kg
C) Flumazenil 0.2 mg
D) Niveles de toxina suero
E) Dilisis urgente
123) Un hombre de 55 aos es admitido al hospital por dificultad respiratoria. Se procede
a colocar tubo endotraqueal y ventilacin mecnica con volumen corriente de 900ml,
frecuencia respiratoria de 12/min y fraccin inspirada de oxgeno del 50%. La presin
positiva al final de la espiracin es de 10cm de agua. Sus medicamentos actuales
incluyen aspirina y heparina subcutnea. Dos das despus, el paciente desarrollo
taquicardia con presin arterial de 110/70mmHg. Examen cardiaco revela mltiples
contracciones prematuras. Su gasometra arterial muestra una pO2 de 40mmHg. Cul
de las siguientes es la causa ms probable de la condicin de este paciente?

45

A) Secreciones bronquiales
B) Arritmia cardiaca
C) Infarto miocrdico
D) Pneumotrax
E) Embolismo pulmonar
124) Un hombre de 43 aos se queja de frecuente sensacin de ardor o quemazn
epigstrica que no se alivia con los anticidos. SE presenta tpicamente cuando debe
levantar objetos pesados en el trabajo y demora 10 a 15 minutos en aliviarse. El niega
asociacin con dolor en cuello o brazos, tos, disnea o dificultad para tragar. Sus
antecedentes incluyen lupus eritematoso sistmico diagnosticado hace 5 aos, para lo
cual toma baja dosis de prednisona diaria. Al examen fsico la presin arterial es de
140/90 mmHg, pulso de 80/min, regular. No hay hallazgos patolgicos al examen de
corazn, pulmones y abdomen. El EKG es normal. Cul de los siguientes es el mejor
prximo paso en el manejo de este paciente?
A) Ecocardiograma
B) Angiografa coronaria
C) Prueba de esfuerzo
D) Prueba de perfusin miocrdica
E) Estudios de motilidad esofgica
F) Endoscopia del tubo digestivo alto
G) Tomografa abdominal
H) Ultrasonido abdominal
I) Tomografa de trax
125) Un hombre de 45 aos llega al hospital con palpitaciones que iniciaron de forma
sbita. Tiene historia de taquicardia supraventricular paroxstica con aproximadamente 3
episodios por ao e historia de asma. Actualmente est utilizando el inhalador beta
agonista. Su presin arterial es 115/75mmHg y su pulso de 160/min. Tiene apariencia
ansiosa. Al examen fsico no hay soplos carotdeo, no hay distensin venosa yugular. El
trax est limpio no hay acropaqua, cianosis o edema. El electrocardiograma revela
taquicardia complejo estrecho. SE realiza masaje carotdeo pero falla en romper el ritmo.
Tratamiento con 6mg IV de adenosina se inician, la cual enlentece el ritmo. 2 minutos
despus el ritmo est en 140/min. Se administra una segunda dosis de adenosina la cual
enlentece una vez ms el ritmo. Tres minutos despus, el pulso est en 140/min, presin
arterial de 110/72mmHg y se le administra la tercera dosis pero falla en el control del ritmo
Cul de los siguientes es el prximo paso en el manejo del paciente?
A) Metoprolol IV
B) Procainamida IV
C) Diltiazem IV
D) Cardioversin sincronizada
E) Cardioversin no sincronizada
126) Un hombre de 45 aos acude a su mdico quejndose de 2 meses de prdida de
peso, debilidad general. Durante este periodo, ha notado un aumento en la pigmentacin
de su piel, polidipsia, poliuria y disminucin del libido. Ha tenido buena salud hasta ahora,
y su revisin mdica hace 1 ao fue normal. Los resultados de laboratorio son: Ferritina
srica 1100 ng/ml; Saturacin de transferrina 55% (normal <45%); Glucosa plasmtica:
170 mg/dl; Alanino aminotransferasa 120 UI/L; Aspartato aminotransferasa: 100 UI/L; Si
46

no recibe tratamiento, este paciente est en riesgo de desarrollar cul de las siguientes
complicaciones?
A) Leucemia mieloide aguda
B) Carcinoma adrenal
C) Tumor cerebral
D) Carcinoma hepatocelular
E) Cncer pancretico
F) Carcinoma de clulas renales
G) Cncer de piel
127) Un hombre caucsico de 53 aos acude a urgencias a las 2AM por dolor severo en
su dedo gordo derecho en las ltimas 2 horas. El se queja que el dolor inici
repentinamente, es sordo, doloroso y empeora en el tiempo a palpitacin severa, no
aliviado con acetaminofn. Sus otros problemas mdicos incluyen hipertensin, diabetes
mellitus tipo 2, hipercolesterolemia y enfermedad lcera pptica. Ha fumado 2 paquetes
de cigarrillos diarios por 30 aos. Bebe 5-6 onzas de alcohol diariamente. Consume
mucha comida chatarra y se ejercita de forma infrecuente. Ambos padre y madre tienen
diabetes e hipertensin. Su hermano mayor muri de infarto miocrdico hace 2 aos. Sus
medicamentos incluyen gliburida, lisinopril, atenolol, simvastatina y famotidina. Su
temperatura es 37.8C, presin arterial de 160/90 mmHg, pulso de 88/min y respiratorio
de 16/min. Al examen, el dedo gordo derecho impresiona marcadamente hinchazo, rojo y
caliente al tacto. A la aspiracin del lquido revela cristales en aguja birefringente
negativo. Adicional a la ya prescrita indometacina oral, cul otra intervencin sera la ms
importante para prevenir futuros ataques de gota?
A) Cesacin tabaquismo
B) Cesacin de alcoholismo
C) Omitir lisinopril
D) Omitir simvastatina
E) Omitir gliburida
F) Omitir famotidina
G) Dieta saludable y ejercicio regular
128) Un hombre de 64 aos es sometido a una reseccin colnica de urgencia por colitis
isqumica extensiva. La ciruga fue sin complicaciones. El paciente recibi piperacilina
tazobactam por los ltimos 5 das. Ha estado en nada por boca desde hace 5 das. Tiene
antecedente de alcoholismo significativo. En el da post operatorio 6 desarrolla sangrado
por el sitio de venopuncin. Su temperatura es de 36.7C, presin arterial es de
121/76mmHg, pulso de 80/min y respiracin de 16/min. Los resultados de laboratorio son:
Hemoglobina: 11.5g/dl; VCM: 88 fl.
Plaquetas: 160,000/mm3, Glbulos blancos:
7,500/mm3,, Neutrfilos: 68%
Bandas : 1%.
Eosinfilos: 1%,
Linfocitos: 24%,
Monocitos: 6%, Tiempo de protrombina: 20 seg (INR = 1.9), Tiempo de trombolastina
parcial activada: 45 seg. No hay antecedentes familiares de importancia. Cul de las
siguientes es la causa ms probable de esta condicin?
A) Prpura trombocitopnica trombtica
B) Sndrome urmico hemoltico
C) Prpura trombocitopnica idioptica
D) Deficiencia vitamnica
E) Coagulacin intravascular diseminada
47

F) Deficiencia de factor VIII


G) Deficiencia de factor IX
H) Enfermedad de Von Willebrand
129) Un hombre de 65 aos acude a urgencias con dolor torcico sub esternal, dificultad
respiratoria, diaforesis que inici repentinamente hace 40 minutos. Desde que inici el
dolor, el paciente ha vomitado en 2 ocasiones. El dolor se irradia al brazo izquierdo y no
remite con nitroglicerina sublingual. El EKG muestra elevacin del ST de 2mm en las
derivadas anteriores. Al examen fsico el paciente tiene temperatura de 36.9C, presin
arterial de 110/70mmHg, pulso de 60/min y respiracin de 32/min con una saturacin de
oxgeno del 90% utilizando oxgeno por cnula nasal a 4L7Min. El examen fsico muestra
un S1 y S2 apagados con presencia de un S3. En los pulmones hay crpitos bibasales
que se extienden hacia la mitad de los campos pulmonares de manera bilateral. Cul de
los siguientes es el mejor prximo paso en el manejo del paciente?
A) Metoprolol
B) Digoxina
C) Quinidina
D) Espironolactona
E) Fluidos IV
F) Verapamilo
G) Amlodipina
H) Furosemida
130) Un hombre de 23 aos que recientemente se someti a trasplante de mdula sea
por leucemia mieloide aguda se queja de fiebre, diarrea severa, prurito e ictericia. La
biopsia heptica muestra marcada disminucin de la cantidad de conductos biliares.
Cul de las siguientes es caracterizada por ese mismo patrn de lesin heptica?
A) Toxicidad por acetaminofn
B) Enfermedad heptica alcohlica
C) Colangitis esclerosante primaria
D) Cirrosis biliar primaria
E) Nutricin parenteral total
F) Hepatitis B crnica
131) Una joven de 28 aos es evaluada por la aparicin reciente de pensamiento
paranoide. Por los ltimos 4 meses, ella ha tenido pensamientos que sus compaeros de
trabajo han estado espindola y hablando a sus espaldas. Ella tiene miedo de ir a
trabajar y se preocupa por perder su empleo. NO ha tenido alucinaciones o depresin y
no hay antecedentes mdicos relevantes. Su padre tiene enfermedad arterial coronaria e
hipertensin y su madre tiene antecedente de depresin. La paciente tiene antecedente
de fumar 9 paquetes / ao de cigarrillos y bebe alcohol solo de forma social. Al examen
fsico no hay hallazgos patolgicos relevantes. Se le inicia olanzapina. Cul de los
siguientes pruebas debe ser la ms importante a realizar luego de 3 meses de estar en
terapia con olanzapina?
A) Nitrgeno de urea y creatinina
B) Hemograma completo
C) Electrocardiograma
D) Glucosa en ayunas y lpidos
48

E) Pruebas de funcin heptica


F) Nivel de prolactina
G) Pruebas de funcin tiroidea
132) Un hombre de 64 aos que no ha visto un doctor en los ltimos 20 aos acude a
urgencias por un dolor torcico agudsimo que inici hace 3 horas de forma repentina. El
describe el dolor como desgarrador en calidad y se irradia a la espalda. Al examen fsico,
usted escucha un soplo decrescendo en la distole temprana a nivel del borde esternal.
La radiografa de trax muestra ensanchamiento del mediastino superior. El EKG es
normal. Cul de las siguientes condiciones mdicas es el probable precipitante de la
presentacin de este paciente?
A) Hipertensin sistmica
B) Sndrome de Marfn
C) Sndrome de Erhlers Danlos
D) Valva artica bicspide
E) Arteritis clulas gigantes
F) Diabetes Mellitus
G) Aterosclerosis
H) Hipertensin pulmonar
133) Un hombre de 43 aos se queja de frecuente sensacin de ardor o quemazn
epigstrica que no se alivia con los anticidos. SE presenta tpicamente cuando debe
levantar objetos pesados en el trabajo y demora 10 a 15 minutos en aliviarse. El niega
asociacin con dolor en cuello o brazos, tos, disnea o dificultad para tragar. Sus
antecedentes incluyen lupus eritematoso sistmico diagnosticado hace 5 aos, para lo
cual toma baja dosis de prednisona diaria. Al examen fsico la presin arterial es de
140/90 mmHg, pulso de 80/min, regular. No hay hallazgos patolgicos al examen de
corazn, pulmones y abdomen. El EKG es normal. Cul de los siguientes es el mejor
prximo paso en el manejo de este paciente?
A) Ecocardiograma
B) Angiografa coronaria
C) Prueba de esfuerzo
D) Prueba de perfusin miocrdica
E) Estudios de motilidad esofgica
F) Endoscopia del tubo digestivo alto
G) Tomografa abdominal
H) Ultrasonido abdominal
I) Tomografa de trax
134) Un hombre de 26 aos acude con historia de 4 semanas de dolor abdominal en los
cuadrantes inferiores tipo intermitente y con calambres acompaado de urgencia rectal,
diarrea sanguinolenta, nuseas y disminucin del apetito. Sus sntomas se han vuelto
ms severos en los ltimos 2 das. No tiene antecedentes mdicos y niega uso de
antibiticos o viajes recientes. Su temperatura es de 38.5C, presin sangunea de
90/50mmHg, pulso de 130/min y respiracin de 16/min. El examen abdominal revela
distencin, dolor difuso a la palpacin y disminucin de los ruidos intestinales. Al examen
rectal muestra dolor rectal y una mezcla de sangre y como en la ampolla. Los laboratorios
se muestran:
49

Hemoglobina: 10.2gr/dl
Leucos: 31,600/mm3
Plaquetas: 459,000/mm3
VES: 54/h
Se le inicia fluidos intravenosos. Cul de los siguientes es el paso ms apropiado en el
manejo de este caso?
A) Enema de Bario
B) Ciruga de urgencia
C) Radiografa abdominal
D) Analgesia con opioides
E) Sigmoidoscopia con biopsia
F) Heces por parsitos y huevos
G) Sulfasalazina y prednisona
135) Un hombre de 64 aos es sometido a una reseccin colnica de urgencia por colitis
isqumica extensiva. La ciruga fue sin complicaciones. El paciente recibi piperacilina
tazobactam por los ltimos 5 das. Ha estado en nada por boca desde hace 5 das. Tiene
antecedente de alcoholismo significativo. En el da post operatorio 6 desarrolla sangrado
por el sitio de venopuncin. Su temperatura es de 36.7C, presin arterial es de
121/76mmHg, pulso de 80/min y respiracin de 16/min. Los resultados de laboratorio son:
Hemoglobina: 11.5g/dl; VCM: 88 fl.
Plaquetas: 160,000/mm3, Glbulos blancos:
7,500/mm3,, Neutrfilos: 68%
Bandas : 1%.
Eosinfilos: 1%,
Linfocitos: 24%,
Monocitos: 6%, Tiempo de protrombina: 20 seg (INR = 1.9), Tiempo de trombolastina
parcial activada: 45 seg. No hay antecedentes familiares de importancia. Cul de las
siguientes es la causa ms probable de esta condicin?
A) Prpura trombocitopnica trombtica
B) Sndrome urmico hemoltico
C) Prpura trombocitopnica idioptica
D) Deficiencia vitamnica
E) Coagulacin intravascular diseminada
F) Deficiencia de factor VIII
G) Deficiencia de factor IX
H) Enfermedad de Von Willebrand
136) Un nio de 2 aos de edad es llevado al mdico por sus padres por rash pruriginoso
que ha sido un problema persistente desde hace 6 meses. El rash es seco, eritematoso y
escamoso e involucra principalmente la cara, la superficie dorsal de las manos y pies. Las
lesiones alternan periodos de remisin y exacerbacin. El tratamiento con esteroides
locales produce alivio temporal. Al examen se revela piel seca con marcas de la piel muy
pronunciadas en las superficies palmares y plantares. La historia familiar es significativa
por rinitis alrgica y asma en varios parientes. El desarrollo del nio es por lo dems
normal. Cul es el diagnstico ms probable?
A) Dermatitis alrgica de contacto
B) Dermatitis atpica
C) Histiocitosis de Clulas de Langerhans
D) Eczema numular
E) Escabiasis
F) Psoriasis
G) Tia corporis

50

137) Una mujer de 62 aos acude para su terapia quimioteraputica por cncer
metastsica de mama. Sin embargo ella se queja de cefaleas leves. Su qumica
sangunea muestra:
Glucosa: 126mg/dl
Sodio: 122mEq/L
Potasio: 4.0 mEq/L
Cloruro: 98 mEq/L
Creatinina: 1.1. mg/dl
Nitrgeno de urea: 16 mg/dl
Cul de los siguientes es el mejor prximo paso para el manejo del paciente?
A) No hacer nada
B) Salina Hipertnica IV
C) Diurticos de asa
D) Diurticos de asa y Cloruro de sodio 0.9% IV
E) Diurticos osmticos
F) Restriccin hdrica
138) Un hombre de 66 aos con historia de hipertensin arterial, diabetes mellitus,
hipercolesterolemia, enfermedad arterial coronaria e insuficiencia renal crnica visita a su
mdico para evaluacin por aumento de su presin arterial. El refiere que su presin
arterial haba estado estable por varios aos pero ha estado elevndose en los ltimos
meses. Su medicacin se ha mantenido igual por los ltimos aos e incluye: aspirina,
metoprolol, clonidina, amlodipina, mononitrato de isosorbide e hidralazina.
A la
evaluacin su presin arterial es de 200/90mmHg, su pulso de 98/min y respiracin de
12/min. A la evaluacin del cuello hay un soplo carotdeo derecho. Los pulmones estn
claros a la auscultacin. La evaluacin Abdominal tambin se revela un soplo. Cul de
los siguientes es el prximo paso ms apropiado para confirmar el diagnstico?
A) Relacin aldosterona renina
B) Ultrasonido doppler duplex
C) Angiografa coronaria
D) Continuar manejo mdico
E) Prueba de estrs
F) Medir niveles de hormona estimulante de la tiroides
G) Centello renal con captopril
H) Angiotomografa computada
139) Un hombre 57 aos diabtico con antecedente de enfermedad de reflujo
gastroesofgico llega a urgencias quejndose de dolor. Hace cuatro horas atrs estaba
sentado, comiendo su comida con familiares, cuando de forma repentina sinti un dolor
severo epigstrico asociado a nusea. Sus signos vitales son temperatura 37.8C
(100F), pulso de 124/minuto y presin arterial 159/95 mmHg. Al examen fsico el paciente
presenta sudoracin excesiva. No hay dolor a la palpacin torcica y los sonidos
respiratorios son claros bilateralmente. Los sonidos cardiacos se auscultan apagados con
ritmo regular. El EKG muestra una elevacin del Segmento ST en las derivadas II y aVF.
24 horas despus de la intervencin coronaria percutnea, el paciente presenta cada de
la presin arterial a 98/52mmHg con pulso en 54/minutos. Cul es el diagnstico ms
probable?
A) Taponamiento cardiaco
B) Falla cardiaca congestiva
C) Ruptura de la pared libre
D) ruptura del msculo papilar
51

E) Infarto del ventrculo derecho


F) Bloqueo de 3er grado
G) Ruptura del septum ventricular
140) Un hombre de 56 aos es llevado a urgencias luego de 10 minutos de prdida de la
conciencia en la tienda de abarrotes. Solo responde al dolor. Su temperatura es de
36.9C (98.4F), presin arterial 110/70mmHg, pulso 116/min, respiracin en 25/min y
saturacin de oxgeno de 86& a oxgeno 100% ventilando por mscara. Las pupilas estn
de igual tamao y reactivas a la luz. La evaluacin cardiolgica revela un S1 y S2 normal,
sin soplos. El abdomen es blando no doloroso. Los gases arteriales son: pH: 7.58; pCO2
en 22mmHg y pO2 en 48mmHg. Cul de los siguientes es el diagnstico ms probable?
A) Hiperventilacin aguda
B) Sobredosis de aspirina
C) Exacerbacin de enfermedad de pulmonar obstructiva crnica debido a bronquitis
D) Embolismo pulmonar
E) Falla renal
F) Sobredosis de sedantes
G) Sepsis
141) Una mujer de 65 aos acude a urgencias debido a debilidad generalizada, mareos y
palpitaciones de 1 da. Ella no ha tenido dolor torcico o dificultad respiratoria. Tiene
antecedente de hipertensin y enfermedad de arteria coronaria. Sus medicamentos
actuales incluyen hidroclorotiazida, aspirina y nitroglicerina. Ella es gerente de oficina
retirada. No bebe alcohol ni fuma cigarrillos. Su temperatura es 37C (98.6F), presin
arterial es 106/65 mmHg, pulso en 130/min, respiracin en 20/min y saturacin de oxgeno
96% a aire ambiente. Los pulmones estn claros a la auscultacin. Evaluacin cardiaca
revela ritmo irregular, sin soplos audibles. El hemograma y panel metablico bsico est
dentro de los lmites normales. El Electrocardiograma se muestra a continuacin.

Cul de los siguientes es el prximo paso ms apropiado en el manejo del caso?


A) Desfibrilacin
B) Amiodarona IV
C) Diltiazem IV
D) Metoprolol IV
E) Marcapaso transcutneo
F) Warfarina
142) Una mujer de 19 aos con historia de trastorno de pnico y ansiedad acude a
urgencias por dificultad respiratoria. Ella niega dolor torcico, palpitaciones, diaforesis o
mareos. Ella ha dejado de tomar sus medicamentos debido a que est convencida de
52

que ya no las necesita. Ella impresiona ansiosa, pero sin distrs agudo. Su temperatura
37C ( 98.6C) presin arterial de 120/70mmHg pulso en 50/minuto y respiracin en
18/min. Su saturacin de oxgeno a aire ambiente es de 99%. A la evaluacin no se
encuentran otras anormalidades. El electrocardiograma se muestra a continuacin:

Se le administra lorazepan y sus sntomas mejoran. Cul de los siguientes es el prximo


paso ms apropiado en el manejo?
A) Administrar amiodarona
B) Anticoagulacin
C) Cardioversin
D) Estudio electrofisiolgico
E) Colocacin de marcapaso
F) Consejera y Educacin.
143) Un hombre de 22 aos acude a urgencias debido a dolor abdominal y vmitos de 1
da de duracin. Sus antecedentes mdicos son impecables y no toma medicacin. No
fuma ni bebe alcohol. No ha tenido contacto reciente con personas enfermas. Su
temperatura es 36.6C ( 97.9F), presin arterial de 112/68mmHg, pulso de 115/min y
respiracin de 24/min con saturacin de oxgeno de 98% a aire ambiente. A la evaluacin
hay una piel seca con disminucin de turgor. Los pulmones estn claros a la
auscultacin. En la parte cardiolgica revela S1 y S2 normales sin soplos. Abdomen
blando, no doloroso con sonidos intestinales normoreactivas. Los laboratorios se
muestran:
pH: 7.28
PaCO2: 26mmHg
PaO2: 95mmHg
Sodio: 135 mEq/L
Cloro: 95mEq/L
Potasio: 5.6 mEq/L
Bicarbonato: 18mEq/L
Cul de las siguientes pruebas es ms probable que resulte anormal?
A) Calcio
B) Creatinina
C) Glucosa
D) Hemoglobina / Hematocrito
E) cido lctico
F) Pruebas de funcin heptica
G) Conteo de clulas blancas
144) Un hombre de 55 aos con antecedente de infarto miocrdico hace 2 aos acude a
urgencias debido a sbita aparicin de falta de aire y severo dolor torcico de 20 minutos.
No tiene sudoracin, nuseas o vmitos. Sus medicamentos actuales incluyen aspirina,
53

atenolol, lisinopril, hidroclorotiazida y lovastatina. Su temperatura es 36.7C (98F),


presin arterial de 110/60mmHg, pulso de 95/ min y respiracin de 18/min. Durante la
evaluacin el paciente deja de responder a estmulos y el electrocardiograma muestra lo
siguiente

Cul de los siguientes es el prximo paso para el manejo?


A) Administrar 1m de Epinefrina IV
B) Administrar un bolo de amiodarona IV
C) Administrar tPA
D) Angioplasta
E) Desfibrilar con 200 Joules
F) Cardioversin elctrica
145) Una madre trae a su hija de 18 meses a urgencias 20 minutos despus de que la
nia tuviera un episodio de atragantamiento, dificultad respiratoria y jadeo mientras
coma unas papas fritas. La madre no recuerda que la nia se pusiera azul durante el
episodio, y la nia volvi a seguir jugando poco despus del episodio con sus muecos.
A la evaluacin, la nia tiene una tos intermitente pero por lo dems est juguetona y no
aparente distres agudo. A la evaluacin fsica es normal, excepto por sibilancias
espiratorias que se notan en los campos pulmonares del lado derecho. La presin arterial
es 95/58mmHg, respiracin de 27/minuto, pulso de 107/minuto y saturando 97% aire
ambiente. La radiografa de trax impresiona normal, pero no se pudieron obtener
radiografas en fase espiratoria. Cul de los siguientes es el mejor prximo paso a
seguir en el manejo?
A) Barrido con los dedos a ciegas
B) Administrar broncodilatador
C) Entubacin endotraqueal
D) Manejo expectante
E) Broncoscopia flexible
F) Broncoscopia rgida
146) Un hombre de 44 aos acude al mdico por disminucin de la destreza manual y
debilidad de sus brazos y piernas en los ltimos 6 meses. La debilidad inici primero en
su brazo izquierdo y luego involucr su brazo y derecho y piernas. La debilidad no mejora
con el reposo. Niega dificultad para hablar, tragar, incontinencia urinaria o intestinal.
Niega cosquilleo o entumecimiento en manos o pies. No tiene antecedentes mdicos de
relevancia. Su madre tiene hipertensin y su padre diabetes mellitus tipo 2. Su
temperatura es 37C (98.6F), presin arterial 134/84mmHg y pulso es 78/min. La
evaluacin muestra espasticidad bilateral y atrofia en las extremidades superiores e
inferiores. La fuerza muscular es 3/5 en las 4 extremidades. Reflejos tendinosos
profundos son hiperactivos, signo Babinsky est presente bilateralmente. La sensibilidad
es normal. Cul de los siguientes es el diagnstico ms probable?
54

A) Esclerosis amiotrfica lateral


B) Evento cerebrovascular hemorrgica
C) Esclerosis mltiple
D) Miastenia Gravis
E) Poliomiositis
F) Polineuropata
147) Un hombre de 60 aos se presenta con quejas dolor extremadamente severo,
pulstil como pistolazo agudo en el lado derecho de la cara. El describe estos episodios
como rayos electricidad que inician cuando toca reas especficas del rostro. Estos
duran 60 segundos y ocurren varias veces durante el da. NO recuerda haber tenido
estos episodios durante la noche. La evaluacin neurolgica no muestra anormalidades.
El rea afecta no puede ser evaluada debido al temor del paciente de desarrollar el dolor.
La resonancia aumentada de gadolinio no muestra anormalidad de nervios craneales.
Cul de las siguientes es el prximo paso ms apropiado en le manejo?
A) Aspirina
B) Carbamazepina
C) Gabapentina
D) Lamotrigina
E) Antiinflamatorios no esteroidales
F) Ablacin esterotctica
G) Descompresin quirrgica del nervio trigmino
148) Una mujer de 34 aos se presenta al mdico con sangre en la orina. Ella refiere
malestar y fiebre en las ltimas 24 horas. Ella presenta respuestas inconsistentes a las
preguntas realizadas. Sus antecedentes mdicos son negativos por enfermedades. No
toma medicamentos, no fuma ni bebe alcohol. A la evaluacin fsica, presin arterial
122/71mmHg, pulso de 89/min. Oxigenando 97% a aire ambiente.
Ligeramente
somnolienta pero se despierta fcilmente. Cabeza y cuello revelan ictericia escleral leve.
No hay distensin venosa yugular. Los pulmones estn claros a la auscultacin, y no hay
soplos, frotes ni galopes. La evaluacin abdominal no muestra ascitis, hepato o
esplenomegalia. Su examen perifrico muestra exantema petequial pero no acropaquia
cianosis o edema. Los valores de laboratorio son:
Glbulos blancos: 11,800/mm3
Hematocrito: 31%
Plaquetas: 75,000/mm3
Sodio: 139 mEq/L
Potasio: 3.7 mEq/L
Cloro: 105 mEq/L
Bicarbonato: 21mEq/L
Nitrgeno de urea: 28 mg/dl
Creatinina: 1.5 mg/dl
Glucosa: 101 mg/dl
TTP: 23 seg
TP: 15 seg
ALT: 24 UI/L
AST: 22 UI/L
Fosfatasa alcalina: 92UI/L
Bilirrubina total: 3.3. mg/dl
Bilirrubina conjugada: 0.4mg/dl
El frotis de sangre perifrica reporta esquistocitos. Cul de los siguientes es la etiologa
ms probable de los sntomas de la paciente?
A) Coagulopata intravascular diseminada
B) Sndrome de Evan
C) Sndrome hemoltico urmico
D) Prpura trombocitopnica inmune

55

E) Enfermedad heptica
F) Prpura trombocitopnia trombtica
149) Una nia de 17 aos es llevado a urgencias debido a cefalea, vmitos y debilidad
progresiva de sus piernas y brazo derecho de 6 das. Ella refiere incontinencia urinaria e
intestinal. Hace una semana, present tos, estornudos, congestin nasal y rinorrea.
Antecedentes personales patolgicos irrelevantes, sin ingesta de medicamentos.
Impresiona letrgica. Temperatura de 38.2C (100.8F), presin arterial de 110/72 mmHg,
pulso de 77/min. Evaluacin neurolgica demuestra debilidad facial derecha. Fuerza
muscular 4/5 en miembro superior derecho y 2/5 en las extremidades inferiores. Hay
pobre tono rectal. Una resonancia cerebral y espinal se realiza mostrando lesiones
hiperintensas a travs de la materia blanca y cordn espinal. Cul de los siguientes es
el diagnstico ms probable?
A) Encefalomielitis Aguda difusa
B) Esclerosis amiotrfica lateral
C) Sndrome de Guillain Barr
D) Polimiositis
E) Leucoencefalopata multifocal progresiva
F) Mielitis transversa
150) Un hombre de 53 aos acude al mdico por 2 episodios de hemoptisis en la ltima
semana. Tiene historia de 2 aos de tos productiva en las maanas de aproximadamente
1 cucharada de esputo blanquesino. Durante la ltima semana la tos se acompaa de una
pequea cantidad de sangre en dos ocasiones. No tiene disnea, fiebre, dolor torcico o
prdida de peso. El paciente ha fumado por los ltimos 30 aos y ha tratado de
disminuirlo recientemente. Trabaja en construccin. Su nico medicamento es ranitidina
para ardor epigstrico. La evaluacin pulmonar los pulmones se auscultan claros. Cul
de los siguientes es la causa ms probable de la hemoptisis del paciente?
A) Bronquiectasias
B) Bronquitis crnica
C) Absceso pulmonar
D) Estenosis mitral
E) Tromboembolismo pulmonar
F) Trombocitopenia
G) Tuberculosis
151) Un hombre de raza blanca acude al mdico para evaluacin para adquirir un seguro
mdico. El generalmente est saludable y no tiene molestias. NO tiene condiciones
mdicas crnicas conocidas. Se ejercita de forma espordica, no fuma pero bebe 6 vasos
de vino por semana. Est casado y tiene un hijo de 9 aos que recientemente estuvo
hospitalizado con esplenomegalia e infeccin por parvovirus. Su padre sufri de anemia
crnica y muri a la edad de 25 aos durante una ciruga para extraerle un rgano
abdominal. Su madre est viva y sana. Su presin arterial es de 125/88mmHg, pulso de
70/min. Examen fsico sin otros hallazgos. Los estudios de laboratorio muestran:
hemoglobina en 12g/dl, hematocrito en 32% y volumen corpuscular medio 88fl y
concentracin de hemoglobina corpuscular media de 39%. Conteo de reticulocitos de
4.2% de clulas rojas y prueba de Coombs negativa. Sus clulas rojas tiene una
sensibilidad aumentada a la lisis en solucin hipotnica. En este momento, Cul de las
siguientes sera el manejo ms apropiado para el caso?
56

A) Iniciar terapia con cido flico


B) Iniciar terapia con hierro
C) Iniciar terapia con corticoides
D) Iniciar terapia con hidroxiurea
E) Enviarlo para esplenectoma
152) Una mujer de 36 aos acude al mdico por visin borrosa y dolor en el ojo izquierdo
de 3 das. El dolor es peor con el movimiento del ojo. Ella no tiene fiebre, historia de
trauma ni epfora. Sus antecedentes mdicos son irrelevantes. No toma medicamentos.
Fuma medio paquete de cigarrillos por da desde hace 15 aos. Su temperatura actual es
de 37.1C (98.8F). La evaluacin muestra una disminucin de la visin en le centro del
campo visual izquierdo y prdida de la visin a color. Hay una respuesta lenta y
disminuida a la luz en el mismo ojo. El disco ptico se ve hinchado, con una hemorragia
en forma de llamas. Cul de los siguientes es la prueba diagnstica ms apropiada?
A) Anticuerpos Anti-Smith
B) Angiografa cerebral
C) Tomografa computada de la rbita
D) Tomografa computada de los senos
E) Resonancia magntica cerebral
153) Una nueva prueba de deteccin de CMV es desarrollada. Tiene una sensibilidad de
90% y especificidad del 80% comparada con el gold estndar. Considere que la prueba
se utiliza en dos poblaciones: La poblacin de las islas del caribe con una prevalencia de
20% y una poblacin en India con una prevalencia de 5%. Cul de las siguientes
aseveraciones siguientes es la ms exacta sobre la nueva prueba?
A) Sensibilidad dela prueba es ms alta en la poblacin del caribe
B) Especificidad de la prueba es mayor en la poblacin de India
C) El valor predictivo positivo de la prueba es mayor en la poblacin del Caribe
D) El valor predictivo negativo es menor en la poblacin de India
E) La prueba no es confiable en la poblacin del Caribe
154) Un hombre de 65 aos acude al mdico por dolor lumbar de 4 meses. El dolor se
irradia hacia la regin gltea y la porcin posterior de sus piernas de forma bilateral. El
dolor es peor cuando camina o se levanta y se alivia al sentarse o caminar cuesta arriba.
Tiene antecedente de 8 aos de hipertensin tratada con amlodipina y lisinopril. Tambin
tiene antecedente de ciruga de reparacin de hernia inguinal hace 3 aos. Su hermano
es hipertenso y su madre tiene diabetes tipo 1. El no fuma cigarrillo pero bebe alcohol de
forma social. Su temperatura es 37C (96.6F), presin arterial de 142/84mmHg, pulso en
80/min. El dolor lumbar es evocado por la extensin de la columna. El paciente pierde el
balance cuando se levanta con sus ojos cerrados. Cul de los siguientes es el
diagnstico ms probable?
A) Sndrome de cauda equina
B) Hernia disco lumbar
C) Estenosis espinal lumbar
D) Enfermedad metastsica
E) Compresin por fractura vertebral

57

155) Una mujer de 50 aos con historia de larga data de enfermedad de Crohn se
presenta al mdico por sensacin de hormigueo y prdida de sensibilidad en las piernas.
Tambin refiere tener problemas para mantenerse equilibrada estando de ie. Estos
problemas han estado ocurriendo en los ltimos meses. Su evaluacin neurolgica es
normal. Su frecuencia cardiaca en reposo es de 100lpm y la presin arterial de su brazo
izquierda sentada es de 100/58 mmHg. El hematocrito se encuentra en 26% y el volumen
corpuscular medio (VCM) en 130fL. El frotis de sangre perifrica muestra anisocitosis,
poiquilocitosis y macroovalocitos con neutrfilos de 6 lbulos. El conteo de leucocitos
total es de 4,000 cel/L y las plaquetas en 140,000/uL. Cul de los siguientes es el
diagnstico ms probable?
A) Anemia aplsica
B) Deficiencia de cobalamina
C) Deficiencia de cido flico
D) Mielodisplasia
E) Mielofibrosis
156) Un hombre de 25 aos acude a urgencias por dolor progresivo en la espalda baja
(rea lumbar) de hace 3 semanas. Tiene intermitentes episodios de sudoracin nocturna,
fiebres y escalofros. Ha tenido dificultad para caminar y controlar sus movimientos
intestinales. No tiene antecedentes mdicos relevantes. Su temperatura es de 38.9C
(102F), presin arterial de 100/60 mmHg y pulso de 120/min. Un soplo sistlico 3/6
eyectivo se irradia a las carotideos desde el borde esternal derecho. Hay dolor a la
palpacin de L4. El conteo de leucocitos es de 32,000 cel /mm3 y la velocidad de
eritrosedimentacin es de 100mm/h. Cul de los siguientes es el microorganismo causal
ms probable?
A) Virus influenza
B) Klebsiella pneumoniae
C) Staphylococcus aureus
D) Streptococcus pneumoniae
E) Streptococcus viridans
157) Un hombre de 32 aos se queja de dolor en su ojo derecho y secrecin. Sus
sntomas iniciaron agudamente en la maana de hoy. El usa lentes de contacto de larga
data y se la ha estado dificultando removerlo en las ltimas semanas. Sus antecedentes
mdicos son importantes por: obesidad, dolor lumbar crnico, asma y enfermedad de
reflujo cido. A la evaluacin hay una secrecin globular amarillenta, espesa en el canto
medial del ojo derecho y en los mrgenes del prpado inferior. La evaluacin del ojo se
muestra en la imagen a continuacin:

58

Cul es el diagnstico ms probable de este paciente?


A) Glaucoma de ngulo cerrado
B) Uveitis anterior
C) Epiescleritis
D) Conjuntivitis pneumocccica
E) Keratitis pseudomnica
F) Hemorragia subconjuntival
158) Una mujer previamente sana de 35 aos acude a atencin mdica debido a parlisis
sbita de las extremidades inferiores y mareo lo cual desarroll desde ayer. Ella ha
estado sufriendo de fiebre, cefalea y rinorrea en los ltimos 5 das. Su queja adicional
nueva incluye visin borrosa y malestar en el ojo izquierdo que ha tenido en las ltimas 2
horas. Su temperatura es de 37C (98.6F), presin arterial de 130/70 mmHg, pulso de
80/min, respiracin de 20/min. Evaluacin neurolgica revela prdida total de fuerza,
reflejos, y sensibilidad en las extremidades inferiores. La fuerza de las extremidades
superiores as como la sensibilidad est conservada. A la evaluacin fsica tambin se
encuentra con una vejiga llena y distendida. Ella est completamente orientada y
consciente. El estudio de resonancia magntica muestra un rea periventricular bien
demarcada de hiperintensidad de T2. Cul de las siguientes es el diangstico ms
probable?
A) Encefalitis por citomegalovirus
B) Sndrome de Guillain Barr
C) Esclerosis mltiple
D) Leucocencefalopata multifocal progresiva
E) Compresin de mdula espinal
159) Un hombre de 39 aos acude a su mdico de atencin primaria por examen de
rutina de salud. El ha estado sintindose bien y no tiene molestias. Ha sido fumador de 2
paquetes de cigarrillo / da en los ltimos 15 aos. Bebe alcohol de forma social y
espordica. Niega uso de drogas ilcitas. Su familia tiene antecedente de cncer de colon
en su padre a la edad de 50 aos que fue tratado con colectoma. Su hermano menor
tuvo cncer de clulas no pequeas pulmonar. Y su madre tiene enfermedad de Graves
en tratamiento. Su abuelo materno falleci de enfermedad de arterias coronarias a la edad
de 71 aos. Cul es el tamizaje o prueba de rastreo ms apropiada para este paciente?

59

A) Evaluar niveles de hormona estimulante de la tiroides


B) Colonoscopia a la edad de 50 aos
C) radiografa de trax por cncer pulmonar
D) Colonoscopia a la edad de los 40 aos
E) Prueba de esfuerzo
F) Citologa de esputo
G) Perfil lipdico cada 6 meses
H) Tomografa de trax y abdomen a la edad de 40 aos
160) Un nio de 10 meses es llevado al pediatra debido a convulsin hace 1 hora. Los
padres refieren que ha estado saludable usualmente hasta esta maana. No ha sido
capaz de alimentarse durante todo el da y est inusualmente somnoliento. Ha vomitado
2 veces en las ltimas 2 horas. Los padres no recuerdan eventos traumticos, sin
embargo la madre refiere que ha estado frustrada en la crianza y que ha tenido que
disciplinarlo con regularidad. El nio no ha tenido enfermedades recientes y ha estado
afebril. Sus antecedentes mdicos son varicela hace 3 meses de la cual se recuper
hace 3 semanas. Es un producto de 34 semanas de gestacin y a la fecha tiene todas
sus inmunizaciones. Ha estado logrando sus hitos de desarrollo sin problemas y no ha
estado en contacto con personas enfermas. A la evaluacin fsica, temperatura 37.2C
(99F), presin arterial de 90/50mmHg y pulso de 110/min. Est somnoliento y con
mnima respuesta a la voz, palpacin o juguetes. Su evaluacin neurolgica no muestra
lesiones focales. Evaluacin cardiovascular, pulmonar y abdominal es normal. Cul de
los siguientes sera el paso ms apropiado en el manejo?
A) Evaluar niveles de amonio por encefalopata de Reye
B) Evaluar niveles de electrolitos y glucosa
C) Realizar puncin lumbar
D) Evaluar fondo de ojo
E) Realizar tomografa computada de cabeza
F) Obtener Electroencefalograma

60

You might also like